Microbiology and Parasitology

June 1, 2018 | Author: Romberg's sign | Category: Virus, Hepatitis B, Bacteria, Hepatitis, Complement System
Share Embed Donate


Short Description

Download Microbiology and Parasitology...

Description

MICROBIOLOGY & PARASITOLOGY 1.

Many of this type of bacteria do not possess superoxide dismutase and catalase A. oblig obligate ate aerob aerobes es B. oblig obligat ate e anaero anaerobe bes s C. facultat facultative ive anaerob anaerobes es D. micr microae oaero rophi philes les p. 66

2.

It is the period when the bacterium is assessing the nutrients present in medium; the bacterium does not divide but there is an intense metabolic activity. The phase of  bacterial growth is A. Lag B. Log C. Stat Statio iona nary ry D. Decli cline p. 53

3.

Whic Which h the the best best to ster steril iliz ize e sci sciss ssor ors s and and othe otherr sha sharp rps? s? A. Glut Glutara aralde ldehy hyde de B. Form Formal alde dehy hyde de C. Iod Iodopho ophor  r  D. Phenol p. 60

4.

These are groups of genes that are clustered together in the DNA of some bacterial species and which may enable them to invade a host A. Pathog Pathogeni enicity city island island B. Episom isomes es C. Tran Transp spos oson ons s D. Repli eplico cons ns p. 98

5.

Which is true about the bacterial chromosome? A. Contain Contains s histones histones that that stabili stabilize ze the DNA DNA B. Composed Composed of a single pair of homologous homologous chromosom chromosomes es C. Replicates Replicates in a very different different manner manner from that that of eukaryotic eukaryotic chromosomes chromosomes D. Contain Contain sing single le copy copy of each each gene gene p. 99

6.

A non-e non-enc ncap apsul sulat ated ed stra strain in of of Hem Hemoph ophilu ilus s infl influen uenza zae e acqu acquire ires s the the gene gene for for caps capsule ule production from a DNA extract of another encapsulated strain. The event that has occurred is A. Tran Transf sfor orma mati tion on B. Conj Conjug ugat atio ion n C. Tran Transd sduc ucti tion on D. Tran Transc scri ript ptio ion n p. 106

1

7.

What is the substance primarily responsible for the structural integrity of the bacterial cell wall and is susceptible to lysozyme? A. Cytop Cytoplas lasmi mic c membran membrane e B. Teic Teicho hoic ic acid acid C. Pept Peptid idog oglyc lycan an D. Lipo Lipopol polys ysacc acchar haride ide p. 22

8.

A hospitalized patient with dysuria and suprapubic pain is treated with ciprofloxacin. What is the mechanism of action of this antibiotic? A. It inhibits inhibits DNA-de DNA-depend pendent ent RNA polym polymeras erase e B. It inhibits inhibits protein synthesis synthesis by binding binding to the the 30S ribosomal ribosomal subunit subunit C. It inhibits protein protein subunits subunits by binding binding to the 50S ribosomal ribosomal subunit D. It inhibits inhibits topoisomerase topoisomerase II (DNA (DNA gyrase) gyrase) p. 164

9.

Which bacterial structure may carry gene for antibiotic resistance? A. Mesos esoso ome B. Peri Peripl plas asm m C. Plasmid mid D. Oute Outerr memb membra rane ne p. 98

10. 10.

What What is the predo predomi mina nant nt indig indigeno enous us flora flora of the the colo colon? n? A. Anaerobic, Anaerobic, gram-negati gram-negative, ve, non-sporenon-spore-forming forming bacteria B. Anaerobic, Anaerobic, gram-pos gram-positive, itive, non-spore-formin non-spore-forming g bacteria bacteria C. Aerobic, gram-negative, gram-negative, non-spore-formi non-spore-forming ng bacteria bacteria D. Aerobic, gram-positive, gram-positive, spore-forming spore-forming bacteria p. 199

11.

Which of the following is the true regarding the bacterial cell? A. He motility motility of bacteria bacteria is due due to flagella, flagella, axial filaments filaments or cilia B. The ability ability to colonize colonize cell surfaces surfaces is is dependent dependent on toxic toxic production production C. The ability to survive in soil for a long period of time is dependent on the production of endospores D. It has the the same ribosome ribosomes s as that of a eukaryotic eukaryotic cells cells p. 35

12.

Which Which of of the the follow following ing is the the count counterp erpart art of mitoc mitochon hondria dria in a bacteria bacteriall cell? cell? A. Cell ell wal walll B. Endo Endosp spor ores es C. Ribos ibosom omes es D. Cell Cell mem membr bran ane e p. 17

13.

Which of the following agents possess either DNA or RNA? A. Viruse ruses s B. Prions C. Bact Bacter eria ia D. Fungi p. 2 2

14.

Most pathogenic microorganism grow best at the temperature range of about _____. A. 15-2 5-20ºC 0ºC B. 30-3 0-37ºC 7ºC C. 50-6 0-60ºC 0ºC D. 80-10 0-100 0ºC p. 65

15.

A 22-year-old woman came in due to meningococcemia. Few hours after the admission, the patient experienced disseminated intravascular coagulation (DIC). The patient shock was due to what component of the causative agent? A. Cell wall B. Capsu psule C. Lipo Lipopol polysa ysacch cchari aride de D. Endo Endosp spor ores es pp. 155-156

16.

Which one of the following antimicrobial agents acts on microorganisms by inhibiting nucleic acid synthesis? A. Stre Strept ptom omyc ycin in B. Cefal efalex exin in C. Cipro Ciproflo floxac xacin in D. Eryt Erythr hrom omyc ycin in p. 189

17.

A newborn was diagnosed with neonatal meningitis. The causative agent was found out to be isolated in the vagina of the mother. Which one of the following microorganisms can be a part of the normal vaginal flora of the mother and can cause neonatal meningitis? A. Mycoplas Mycoplasma ma pneumo pneumonia niae e B. Stap Staphyl hyloco ococc ccus us aureus aureus C. Esch Escheri erich chia ia coli coli D. Strepto Streptococc coccus us agalac agalactia tiae e p. 199

18.

Which of the following is the “C3 activation unit” in the classical complement pathway? A. C1q B. C3 C. C4b, C2 C2a D. C5, C5, C6, C6, C7, C7, C8 and and C9 C9 p. 136

19.

The classic pathway of the complement system is activated when__  A. The Fab portion of of IgM binds binds to the C1s C1s of the complement complement pathway pathway B. The Fc portion of of IgE binds binds to the C1q C1q of the complement complement pathway pathway C. The hypervariabl hypervariable e regions regions of the heavy and light light chains binds binds to the membrane of the bacterial cell D. The mannose mannose binding binding lectin stimulates stimulates mannose mannose associated associated serine serine protease p. 136

3

20.

Antigen-presenting cells that activate helper T cell must express which one of the following on their surfaces? A. IgE B. Gamm Gamma a inter interfer feron on C. Class Class I MHC MHC antige antigens ns D. Class Class II MHC antigen antigens s p. 139

21.

Increased susceptibility with Hepatitis virus and Candida albicans would be expected with a deficiency of which of the following cells? A. PMN B. Eosi Eosino noph phil ils s C. B ce cells D. T cells lls p. 138

22.

In Type I hypersensitivity, the predominant antibody that is elicited is: A. IgA B. IgE C. IgG D. IgM p. 140

23.

Cont Contac actt der derma mati titi tis s is an exam exampl ple e of: of: A. IgE IgE medi mediat ated ed B. Comp Complem lemen entt mediat mediated ed C. IgG IgG med media iate ted d D. Cell Cell med media iate ted d p. 141

24. 24.

This This immu immunog noglob lobuli ulin n is impo importa rtant nt in prev preven entin ting g diarr diarrhea hea in in the newb newborn orn A. IgA B. IgM C. IgE D. IgD p. 129

25. 25.

Main Main comp compon onen entt in seco second ndar ary y resp respon onse se.. A. IgA B. IgM C. IgG D. IgD p. 135

26. 26.

At birth, birth, the the main main sourc source e of of Hem Hemato atopoe poetic tic Stem Stem Cell Cell is is the the A. Liver  B. Spleen C. Bone Bone Marr Marrow ow D. Thym hymus p. 126 4

27. 27.

Which Which meth method od will will you you see see to detect detect antig antigen ens s dire directl ctly y from from tiss tissue? ue? A. ELISA B. Ouch Ouchter terlon lony y techn technic ic C. Immun Immunof ofluo luores resce cenc nce e D. Comp Complem lemen entt fixati fixation on p. 142

28.

Which is true about humoral immunity and antibodies? A. Antibodies Antibodies found found on the the surface surface of B cells are IgA B. IgG has has two bindings bindings sites for an antigen C. IgM is the the main protection protection of the newborn newborn up to 6 (six) months D. The Fab Fab region region of IgE IgE binds binds to mast mast cell cell receptor  receptor  p. 128

29. 29.

Which Which of of the the follo followi wing ng com comple pleme ment nt cas cascad cade e does does not not inv involv olve e C3b? C3b? A. C5 conver convertase tase of of the alterna alternativ tive e pathway pathway B. C5 conver convertase tase of the class classic ic pathway pathway C. C3 convertas convertase e of the alternative alternative pathway pathway D. C3 conver convertase tase of of the classic classic pathw pathway ay p. 136

30.

Which of the following cells is a part of the innate immune response? A. B-ly B-lymp mpho hocy cyte tes s B. T-he T-helpe lperr lympho lymphocyt cytes es C. T-cytot T-cytotoxic oxic lymphocy lymphocytes tes D. Macr Macrop opha hage ges s p. 123

31.

Which of the following is LEAST associated with Enterohemorrahgic E. coli? A. ferment fermentatio ation n of of sorbit sorbitol ol B. stra strain in 015 0157: 7:H7 H7 C. underco undercooked oked hamburg hamburgers ers D. hemolyt hemolytic-u ic-uremi remic c syndro syndrome me p. 253

32. 32.

Which Which of of the the follo followi wing ng is true true abo about ut chole cholera ra and and its its caus causat ative ive agen agent? t? A. A secretory secretory diarrhea diarrhea due to increases increases in cGMP in the intestinal intestinal cells cells B. The stool stool is described described as ‘rice water-like’ water-like’ C. The causative causative agent is non-motile, non-motile, nonfermenta nonfermentative tive D. Grows Grows poorly poorly in medium medium with with alkalin alkaline e pH p. 271

33.

The appearance of moistpale papules or condylomas in the anogenital area is characteristic of  A. Prim Primary ary syphi syphilis lis B. Seco Second ndary ary syphil syphilis is C. Terti Tertiary ary syphil syphilis is D. T. pall pallid idum um p. 332

5

34.

A 4-year old Asian child develops an infection with Chlamydia trachomatis. How does infection with this organism cause blindness? A. Retin Retinal al deta detachm chment ent B. Catar Catarac actt form formati ation on C. Hemorrh Hemorrhage age of the anteri anterior or chamber  chamber  D. Scarrin Scarring g of of the the corne cornea a p.360

35. 35.

Whic Which h is asso associ ciat ated ed wit with h ath ather eros oscl cler eros osis is? ? A. K. pneu pneumo moni niae ae B. C. tra trach chom omat atis is C. S. pyog pyogen enes es D. C. pne pneum umon onia iae e p.364

36.

Which Which of of the the followi following ng is LEAS LEAST T effec effective tive against against M. pneum pneumonia oniae e infect infection ion? ? A. Eryt Erythr hrom omyc ycin in B. Tetr Tetrac acyc ycli line ne C. Kana Kanamy myci cin n D. Vanc Vancom omyc ycin in p. 345

37.

The only disease that can be diagnosed by demonstration of spirochetes in peripheral blood smear  A. Relap Relapsin sing g fev fever  er  B. Lept Leptop opir iros osis is C. Pinta D. Lyme Lyme dise diseas ase e p. 335

38. 38.

Which Which of of the the follo followi wing ng is LEA LEAST ST ass assoc ociat iated ed with with Heli Helico copte pterr pylor pylori? i? A. It requires requires at least least 2% NaCl NaCl for growth (halophil (halophilic) ic) B. It requires requires increased increased carbon carbon dioxide dioxide tension and decreased decreased oxygen oxygen tension C. It is sensitive sensitive to the acidity of the gastric juice D. Most commonly commonly diagnosed diagnosed by the demonstra demonstration tion of urease urease production production p.275

39. 39.

In whic which h type type of diar diarrhe rhea a is the the use use of anti antibio biotic tics s not not recom recomme mende nded? d? A. Salmo Salmone nello llosis sis B. Campylo Campylobact bacter er infectio infection n C. Chole olera D. Shig Shigel ello losi sis s p. 259

6

40.

Which causes urinary tract infection characterized by alkaline urine and formation of  urinary calculi? A. Shige Shigella lla dyse dysente nteria riae e B. Esch Escheri erich chia ia coli coli C. Prot Proteu eus s mirabi mirabili lis s D. Enter Enteroba obacte cterr cloac cloacae ae p. 254

41.

Which is LEAST acceptable as specimen for the diagnosis of anaerobic infection A. Trans Transtra trach chea eall aspir aspirate ate B. Supra Suprapu pubic bic asp aspira irate te C. Vagi Vagina nall swab swab D. Sample Sample of tissue tissue from from infect infected ed site site p. 309

42.

Which of the following microorganisms takes up the color of Crystal violet upon properly performed Gram staining? A. Neiss Neisser eria ia gonorr gonorrhea heae e B. Clostri Clostridiu dium m perfr perfring ingens ens C. Esch Escheri erich chia ia coli coli D. Pseu Pseudom domon onas as sp. p. 208

43.

Which of the following is a Toxoid vaccine? A. Pnue Pnuemo moco cocc ccal al B. Measles C. Vari Varice cell lla a D. Diph Diphth ther eria ia p. 215

44. 44.

The The mos mostt inf infec ecti tiou ous s sta stage ge in pert pertus ussi sis s is: is: A. Catar Catarrha rhall sta stage ge B. Prod Prodom omal al sta stage ge C. Paro Paroxys xysma mall stag stage e D. Conva Convales lescen centt stage stage p. 283

45.

Regarded as a TB marker in the diagnosis of tuberculous effusion: A. Gamma Gamma inter interfer feron on B. AFB AFB smea smear  r  C. Casea Caseatio tion n necros necrosis is D. PPD PPD posit positivi ivity ty p. 407

46.

Presence of immunity to C. diphtheriae is indicated by: A. Posi Positiv tive e ele elek k test test B. Negat Negative ive elek elek test test C. Posi Positiv tive e schick schick test test D. Negat Negatice ice sch schick ick test test

7

p. 215 47. 47.

Which Which of the the A. B. C. D.

foll follow owing ing is MOS MOST T like likely ly acqu acquire ired d from from cave caves? s? Blas Blasto tomy myco cosi sis s Histo Histopl plas asmos mosis is Cocci Coccidi dioid oidom omyco ycosis sis Paracoc Paracoccidi cidioid oidomyc omycosis osis

p. 642 48.

Asteroid bodies are seen tissue infected with A. P. verr verruc ucos osa a B. P. bo boydii C. S. sche schenc ncki kiii D. L. lo loboi boi p. 634

49.

The agents of chromomycosis are seen in infected tissues as A. Intra Intrace cellu llular lar yeast yeast B. Gran ranules les C. Scle Sclero rotic tic bodi bodies es D. Cigar Cigar-s -shap haped ed bodies bodies p. 635

50.

Which of the following is NOT dimorphic? A. P. bo boydii B. H. caps capsul ulat atum um C. S. sche schenc ncki kiii D. C. imm immitis itis p. 636

51. 51.

The The agent agent of A. B. C. D.

blas blastom tomyco ycosis sis are seen seen in in infe infecte cted d tissu tissues es as as Intra Intrace cellu llular lar yeast yeast Single Single buddin budding g yeast yeast Multiple Multiple budding budding yeast yeast Cigar Cigar-s -shap haped ed bodies bodies

p. 642 52.

The physician closely monitored the patient’s serum creatinine and ion levels. The patient is taking certain anti-fungal drug. What possible antifungal agent may affect the renal function of this patient? A. Keto Ketoco cona nazo zole le B. Terb Terbin inaf afin ine e C. Gris Griseo eofu fulv lvin in D. Amph Amphot oteri ericin cin B p. 653

53.

The primary site of echovirus multiplication in the human host is A. The The musc muscula ularr syste system m B. The The alimen alimentar tary y tract tract C. The anterior anterior horn cells cells D. The respirat respiratory ory system system

8

p. 487 54.

Which of the following properties of polioviruses is not shared by rhinovirus? A. Icosa Icosahed hedral ral symm symmet etry ry B. Resista Resistance nce to lipid lipid solvent solvents s C. Nake Naked d viru virus s D. Stabili Stability ty at acid acid pH pH (pH3.0 (pH3.0)) p. 489

55. 55.

Segm Segmen ented ted,, double double-st -stran randed ded RNA RNA geno genome me tha thatt is a com commo mon n caus cause e of  viral gastroenteritis? A. Astr Astrov ovir irus us B. Norw Norwal alk k vir virus us C. Cali Calici civi viru rus s D. Rota Rotavi viru rus s pp. 505-507

56.

Diagnost Diagnostic ic of of Congen Congenital ital rubella rubella is the the demo demonstr nstratio ation n of rubella rubella antibod antibodies ies like: like: A. Mate Matern rnal al IgM IgM B. Neon Neonat atal al IgM IgM C. Neon Neonat atal al IgA IgA D. Mate Matern rnal al IgA IgA p. 568

57. 57.

A vira virall cause cause of neph nephro ropat pathy hy in in immu immunoc nocom ompr prom omise ised d patie patient nt is is A. Human Human papill papillomav omavirus irus,, all types types B. Human Human papillom papillomavir avirus, us, low-ris low-risk k types C. Hepa Hepati titi tis sC D. Polyo Polyomav maviru irus s BK BK pp. 598-599

58.

Which Which of of the the follow following ing individu individuals als may be at at incre increased ased risk of acqui acquiring ring and HIV infection? A. A pregnant pregnant mothe motherr with a seafarer seafarers s husband husband B. A secreta secretary ry at an AIDS AIDS instit institute ute C. A doctor doctor with a colleague colleague that that is HIV HIV positive positive D. A male celebrity celebrity who has multiple sexual sexual partner partner including including a prostitute pp. 616-617

59.

Type of papillomavirus that is commonly associated with cervical carcinoma A. Type 1 B. Type 6 C. Type 9 D. Type 18 18 p. 601

60.

Progressive Multifocal leukuencepalopathy is associated with a virus that is described as __. A. ssDNA, ssDNA, icosahe icosahedral dral,, nake naked d B. dsDNA, dsDNA, helical, helical, envelope enveloped d C. dsDNA, dsDNA, icos icosahe ahedral dral,, naked naked 9

D. ssRNA, ssRNA, segment segmented, ed, naked naked pp. 597-599 61.

Which of the following virus that can enter a skin through abrasions? A. Aden Adenov ovir irus us B. Poly Polyom omav avir irus us C. Papi Papillo llomav maviru irus s D. Rhin Rhinov ovir irus us p. 599

62.

A 6-year old child had recently had fifth disease. Her 43-year old mother subsequently developed a rash and arthalgia. Which one of the following best describes the causative agent? A. It has has the same same size size as as poxviru poxvirus s B. It has a helic helical al symmetr symmetry y C. Inact Inactiva ivated ted by by ether  ether  D. A single single-str -strand anded ed DNA DNA genome genome pp. 414-416

63.

HIV HIV is is a ret retro rovi viru rus. s. A ret retro rovi viru rus s is is a A. Cause Cause tumo tumors rs in in mouse mouse B. Conta Contain in a DNA DNA geno genome me C. Cause rapidly progressive progressive neurologic neurological al disease disease D. Multipl Multiplies ies in in the the nucleu nucleus s p. 605

64. 64.

Most Most com commo mon n caus cause e of pos postt tran transf sfus usio ion n hepa hepati titi tis s A. HAV B. HBV C. HCV D. HDV p. 466

65.

Site of latent infections is at the sacral ganglia: A. HSV 1 B. HSV 2 C. EBV D. CMV pp. 433-434

66.

Which of the following viruses can establish a latent infection? A. Poxv Poxvir irus uses es B. Rube Rubell lla a virus viruses es C. Herpe Herpesv sviru iruses ses D. Coron Coronav aviru iruses ses p. 371

67.

A 23-year-old medical student experienced fever, sore throat, and lymphadenopathy. These were accompanied by lymphocytosis and atypical cells in the blood smear. True statement regarding the causative agent A. the major major target target cell of the virus is the T lymphocyte lymphocyte B. the agent agent multi multiplie plies s in the cytop cytoplasm lasm C. it can be be transmitted transmitted thru direct contact contact with with the lesion lesion 10

D. it possess possess a double-strand double-stranded ed DNA DNA genome genome p. 429 68.

The following hepatitis B marker is indicative of active viral replication A. HBsAg B. HBeAg C. Anti Anti-- HBs HBs D. Anti Anti-- HB HBe p. 468

69.

The habitat is the large intestine. A. Entam Entamoe oeba ba gingiv gingivali alis s B. Entamoe Entamoeba ba histoly histolytica tica C. Giar Giardia dia lambli lamblia a D. Naeg Naegler leria ia fowle fowleri ri p. 104

70.

The infective stage of Entamoeba histolytica to man has A. Pseu Pseudo dopo podi dia a B. Inge Ingeste sted d red blo blood od cells cells C. Bull Bull’s ’s eye eye karyo karyoso some me D. Pointed Pointed chromat chromatoida oidall bodies bodies pp. 103-104

71.

The most commonly involved area in Amoebiasis is the A. Brain B. Cecum C. Liver  D. Recto Recto-si -sigm gmoid oid region region pp. 108-109

72.

There is no increase in the number of nuclei of  A. Balan Balantid tidiu ium m coli coli B. Enta Entamo moeb eba a coli coli C. Enta Entam moeba oeba D. Giar Giardia dia lambli lamblia a p. 215

73.

Tissue invasion is relatively slow and tends to stimulate granuloma formation A. Acanth Acanthamoe amoeba ba caste castellan llanii ii B. Balan Balanti tidiu dium m coli coli C. Entamo Entamoeba eba histolyt histolytica ica D. Naeg Naegler leria ia fowle fowleri ri p. 142

74.

Mosquito vector of Malaria in the Philippines is A. Aede Aedes s aeg aegyp ypti ti B. Anophe Anopheles les flaviro flavirostri stris s C. Culex Culex quinquef quinquefasci asciatus atus D. Manso Mansoni nia a annula annulata ta

11

p. 164

75.

There is secondary exo-erythrocytic phase in A. Benign Benign tertian tertian malaria malaria B. Maligna Malignant nt tertian tertian malaria malaria C. Quar Quarta tan n mala malaria ria D. Sub-t Sub-tert ertian ian malar malaria ia p. 177

76.

The duration of the Erythrocytic-Schizogonic Cycle is 72 hours. A. A. Benign Benign tertian tertian malaria malaria B. Maligna Malignant nt tertian tertian malaria malaria C. Quart Quartan an malari malaria a D. Sub-t Sub-tert ertian ian malar malaria ia pp. 184-191

77.

The gametocytes of the organism are described as banana or crescent-shaped. A. Plasmod Plasmodium ium falcipa falciparum rum B. Plas Plasmo modiu dium m malar malariae iae C. Plas Plasmo modiu dium m oval ovale e D. Plas Plasmo modiu dium m viva vivax x pp. 188-190

78.

Produces rectal prolapse in heavy infection especially among children A. Capilla Capillaria ria philippi philippinen nensis sis B. Trich Trichine inella lla spir spirali alis s C. Trch Trchuri uris s trichiu trichiura ra D. Ascaris Ascaris lumbrico lumbricoides ides p. 243

79.

Bachman intradermal test & Xenodiagnosis are use for the diagnosis of infection cause by A. Capilla Capillaria ria philippi philippinen nensis sis B. Trich Trichin inel ella la spira spirali lis s C. Trch Trchuri uris s trich trichiur iura a D. Ascaris Ascaris lumbrico lumbricoides ides p. 239

80.

The most common cause of Greeping Eruption or Cutaneous Larva Migran A. Ancytos Ancytostoma toma brazilie braziliense nse B. Enterob Enterobius ius vermicul vermicularis aris C. Strongy Strongyloid loides es sterc stercoral oralis is D. toxo toxoca cara ra cani canis s p. 281

81. 81.

The The most most com commo mon n caus cause e of Visc Viscer eral al Lar Larva va Mig Migra ran n A. Ancylos Ancylostoma toma brazilie braziliense nse B. Enterob Enterobius ius vermicul vermicularis aris C. Strngyl Strngyloid oides es stercola stercolaris ris D. Toxo Toxocar cara a cani canis s 12

p. 325 82.

Produces autoinfection to man A. Ancy Ancylos lostom toma a duode duodena nale le B. Asca Ascaris ris lumb lumbric ricoid oides es C. Necat Necator or ameri america canus nus D. Strong Strongyloi yloides des ster stercola colaris ris p. 256

83.

What parasite produces Iron deficiency anemia to man? A. Asca Ascaris ris lumb lumbric ricoid oides es B. Necat Necator or americ americanu anus s C. Strongy Strongyloid loides es sterc stercolar olaris is D. Trich Trichur uris is trichi trichiura ura p. 284

84.

Associated with Noctrunal Pruritus Ani. A. Asca Ascaris ris lumb lumbric ricoid oides es B. Enterob Enterobius ius vermicul vermicularis aris C. Necat Necator or ameri america canus nus D. Trich Trichur uris is trichi trichiura ura p. 304

85.

Graham Scotch tape technique is used for the diagnosis of the infection caused by A. A. Asca Ascaris ris lumb lumbric ricoid oides es B. Enterob Enterobius ius vermicul vermicularis aris C. Necat Necator or ameri america canus nus D. Trich Trichur uris is trichi trichiura ura p. 305

86. 86.

What What para parasi site te suck sucks s and and inge ingest sts s blo blood od? ? A. Ancylos Ancylostoma toma duoden duodenale ale B. Asca Ascaris ris lumb lumbric ricoid oides es C. Enterob Enterobius ius vermicul vermicularis aris D. Trich Trichur uris is trichi trichiura ura p. 284

87.

What parasite shows Tramway sign in the X-ray? A. Ascari Ascaris s lumbrico lumbricoide ides s B. Enterob Enterobius ius vermicul vermicularis aris C. Necat Necator or ameri america canus nus D. Trich Trichur uris is trichi trichiura ura p. 317

88.

The infective stage of the parasite is found in crabs or crayfishes A. Clon Clonorc orchis his sine sinens nsis is B. Echi Echinos nostom toma a ilocan ilocanum um C. Fasc Fasciol iola a hepat hepatica ica D. Parago Paragonimu nimus s westerm westermani ani p. 465 13

89.

What parasite is associated with neoplasm of the biliary duct or cancer of the liver? A. Clon Clonor orch chis is sinens sinensis is B. Fasci Fasciola ola hepa hepatic tica a C. Opistho Opisthorchi rchis s felineus felineus D. Paragon Paragonimus imus westerm westermani ani p. 477

90.

What parasite produces infection that stimulates tuberculosis? A. Clon Clonorc orchis his sine sinens nsis is B. Fasci Fasciola ola hepa hepatic tica a C. Opistho Opisthorchi rchis s felineus felineus D. Parago Paragonimu nimus s westerm westermani ani pp. 467-468

91.

What is the infective stage of Schistosoma to man? A. Cerca ercari riae ae B. Embry Embryon onate ated d egg egg C. Meta Metace cerc rcia iae e D. Mira Miraci cidi dium um p. 417

92. 92.

Mollu Mollusc scan an host host of of Schist Schistos osom oma a in in the the Phili Philipp ppine ines s is A. Austr Australo alorb rbis is mysta mystax x B. Biomph Biomphalar alaria ia alexandr alexandrina ina C. Coxie Coxiella lla bur burnet netiiii D. Oncomel Oncomelani ania a quadras quadrasii p. 416

93.

Laboratory procedure that makes use of the egg in the test is A. CFT B. CHR C. COPT D. Caso Casoni ni test test p. 770

94.

The infective of Diphyllobothrium latum to man to produce adult infection is the A. Cysti Cystice cerco rcoid id larv larva a B. Cysti Cystice cercu rcus s larv larva a C. Proc Procerc ercoid oid larva larva D. Sparg Spargan anum um larva larva p. 497

95.

Pro Produce duces s Ocul Ocular ar cys cysti tice cerc rcos osis is to to man man A. Dipyl Dipylidi idium um can caninu inum m B. Taen Taenia ia sagi sagina nata ta C. Taen Taenia ia soli solium um D. Hyme Hymeno nolep lepis is nana nana p. 518 14

96.

Lateral uterine branches in gravid proglottid are counted for diagnosis of infection cause by A. Diphy Diphyllo llobo bothr thrium ium latum latum B. Dipyl Dipylidi idium um can caninu inum m C. Hyme Hymeno nolep lepsis sis nana nana D. Tean Teania ia sagin saginata ata p. 522

97.

Taenia saginata is associated with A. Cat B. Cattle C. Dog D. Pig p. 521

98. 98.

Ultra Ultraso soun und d and MRI MRI usua usually lly hel help p in the the diagn diagnosi osis s of the the infect infection ion cau cause se by A. Dipyl Dipylidi idium um can caninu inum m B. Echino Echinococ coccus cus granu granulos losus us C. Hymeno Hymenolepis lepis diminuta diminuta D. Hyme Hymeno nolep lepis is nan nan p. 532

99.

This organis organism m produc produces es a blue-gr blue-green een pigm pigment ent and and there therefore fore produce produces s a char characte acterist ristic ic blue-green pus. A. Stap Staph h aure aureus us B. Pseudo Pseudomona monas s aerug aerugino inosa sa C. Aspergi Aspergillus llus fumigat fumigatus us D. Strep Strep.. pyoge pyogenes nes Pseudomonas aeruginosa produce  pyocyanin pigment w/c is blue green in color and also has antibiotic-like characteristics against some bacteria Reference: Zinsser p. 577

100.

Each of the following following statements statements concerning concerning the surface structures structures of bacteria is correct correct EXCEPT: A. Pili mediate mediate the interactio interaction n of bacteria bacteria with mucosal epithelium B. Polysaccharides Polysaccharides capsules retard phagocytosis phagocytosis C. Both gram negative rods rods and cocci have have polysaccharid polysaccharides es (endotoxin (endotoxin in their  cell wall) D. Bacterial Bacterial flagella flagella are not antigenic antigenic in humans humans Bacterial flagella are made of proteins and are useful in distinguishing serovars or variation of gram negative bacteria such as E. coli H. antigens Sal. H .antigens. Reference: Zinsser. Chapter 3

101. 101.

The most most common common site site of asymp asymptoma tomatic tic gonoco gonococca ccall infection infection in in women women is the: A. B. C. D.

Vagina Myom Myomet etri rium um Ureth rethrra Endo Endoce cerv rvix ix 15

The columnar epithelial cells here are the ones that are being infected by N. gonorrhea.(A). Vaginal walls composed of stratified squamous epithelial cells which are not colonized. (B) and (C) are not affected. Reference: Zinsser p. 456 102. 102.

The follo following wing etiologic etiologic agents agents are known known to cause cause diarrh diarrhea ea EXCEP EXCEPT: T: A. Clostrid Clostridium ium perfring perfringens ens B. Rotav otavir irus us C. V. chol choler era a D. Prot Proteu eus s vulg vulgari aris s Protues vulgaris produce urinary tract infection. C. clostridium perfringens diarrhea thru it’s invasiveness so blood and mucus is (+) V. cholera-produced diarrhea by virtue of enterotoxin. Rotavirus most common cause of viral gastroenteritis in small children. Reference: Zinsser p. 552

103. 103.

A 20 year year old old male male develo developed ped a carbunc carbuncle le with with surrou surroundin nding g celluli cellulitis. tis. Gram Gram stain stain revealed gram positive cocci, catalase and coagulase positive. Which of the following antibiotics is initially appropriate for this patient? A. Vanc Vancom omyc ycin in B. Clox Cloxac acil illi lin n C. Cefr Cefria iaxo xone ne D. Peni Penici cill llin in 90% of S. aureus strains are resistant to penicillin because pf production of  plasmid-derived β-lactamses. Such organisms can be treated with βlactamase-resistant penicillin such as cloxacillin or dicloxacillin oxacillin. Oxacillin and Nafcillin have unpredictable absorption by oral route, thus is not recommended Reference: Zinsser p. 412

104. 104.

Which Which of the followi following ng features features help help disting distinguish uish group group A streptoc streptococcu occus s from other  other  streptococcal species? A. Bacitra Bacitracin cin susc suscept eptibil ibility ity B. Bile Bile solu solubi bili lity ty C. Optochin sensitivity D. Growth Growth inhibiti inhibition on in 6.5 % NaCl NaCl Group A streptococcus is a β-hemolytic streptococcus and can usually be distinguished from the other β-hemolytics by its bacitracin susceptibility or by precipitation with specific anti sera (against its "Lancefrield antigen"). Bile solubility test and optochin test distinguishes the L-hemolytic streptococci, S. pneumoniae is bile soluble and and optochin sensitive. Among the hemolytic streptococci, S. viridans growth is inhibited by 6.5% NaCl while Enterococcus species grows in 6.5% NaCl. Reference: Zinsser p. 419

105. 105.

Which Which of of the the follow following ing Clostrid Clostridia ia caus cause e gangre gangrene? ne? A. Clost Clostrid ridium ium tetani tetani B. Clostridium botulinum C. Clostri Clostridiu dium m perf perfring ringens ens D. Clost Clostrid ridium ium diffici difficile le C. tetani causes lockjaw. C. botilinum causes botulism. C. difficile causes antibiotic associated pseudomembranous colitis. C. perfringens cause gasgangrene and Food poisoning. 16

Reference: Zinsser p. 639 106. 106.

This gram gram positiv positive e rod is usually usually associa associated ted with with bacteria bacteriall vaginosis vaginosis produc producing ing malodorous vaginal discharge and "clue cells" A. Erysipel Erysipelothr othrix ix rhusopat rhusopathiae hiae B. Lactobacillus acidophilus C. Gardenel Gardenella la vaginal vaginalis is D. Propion Propionibac ibacteri terium um acnes acnes E. rhusopathiae causes erysipeloid skin infection that resembles erysipelas occurring among meat and fish handlers. L. acidphilus is normal normal flora of  genital tract and is the main source of lactic acid. P. acnes produces lipase which contributes to the genesis of acne. Reference: Zinsser p. 604-605

107. 107.

Which Which of the followi following ng organism organisms s is MOST likely likely to involve involve invasi invasion on of the intestin intestinal al mucosa. A. Vibr Vibrio io cho chole lera ra B. Shigella sonnei C. Enterot Enterotoxig oxigenic enic E. coli D. Clostrid Clostridium ium botulinu botulinum m Shigella sonnei is the only invasive, presence of blood and mucus in the stool. Clostridium botilinum , V. cholera and Enterotoxigenic E. coli causes diarrhea by producing enterotoxin Reference: Zinsser p. 561

108.

Food poisoning that produces gastrointestinal gastrointestinal symptoms symptoms approximat approximately ely 1-2 1-2 hours hours after  eating is most likely to be due to: A. Salm Salmon onell ella a enter enteriti itidis dis B. Camp Campyl yloba obacte cterr jejuni jejuni C. Clostrid Clostridium ium perfring perfringens ens D. Stap Staph. h. aur aureu eus s Staph aureus – the incubation period is 1-2 hrs. after ingestion of  contaminated food. Sal. enteritidis – 6 hrs. – 2 days. Campylobcacter – 8 hrs. – l day. C. perfringens – 8 hrs. – 12 hrs. Bacillus aureus – 8 hrs. – 12 hrs. V. cholera – 2 – 3 days. V. parahaemolyticus – 8 hrs. – 2 days Reference: Zinsser p. 412

109. 109.

A 30 year year old pregnan pregnantt woman woman had a normal normal delive delivery ry but the the fetus fetus was stillb stillborn. orn. CSF CSF examination showed positive rods. The pathogen is most likely: A. Listeri Listeria a monocyt monocytoge ogenes nes B. Gardenerella vaginalis C. Group Group B Strep Streptoco tococcu ccus s D. Staphylo Staphylococc coccus us aureus aureus Listeria monocytogenes is a gram positive rod and can infect pregnant women and poses the threat of stillbirth or serious damage to the fetus and also cause neurological disease. Gardenella vaginalis gives rise to frothy, fishy odor vaginal discharge, it can also cause premature births and low-birthweight infants. Reference: Zinsser 484

17

110.

Which of these these organisms organisms is a major pathogen for nosocomia nosocomiall infections infections because because of its ubiquitous present in the hospital environment? A. Stap Staph. h. Aure Aureus us B. Stap Staph. h. Epid Epiderm ermidi idis s C. Serra Serratia tia marces marcesce cens ns D. Pseudo Pseudomona monas s aerog aerogino inosa sa Pseudomonas aeruginosa, S. aureus - yellow gold pigment when cultured at room temperature. S, epidermidis - white pigment when cultured at room temperature. Serratia marcescens - red pigment Reference: Zinsser p. 577

111.

An individual individual experiences experiences severe severe diarrhea after eating eating sushi sushi (raw (raw fish) fish) in a West West Coast Coast restaurant. The most probable etiologic agent is: A. Yersenia Yersenia enteroc enterocolit olitica ica B. Salmonella enteritidis C. Shige Shigella lla sonnei sonnei D. Vibrio Vibrio parahae parahaemoly molyticu ticus s Vibrio parahaemolyticus - is a salt loving organism and found in fishes. Salmonella enteritidis - found in chicken, pig. Yersenia enterocoliticareserviors rodents Reference: Zinsser p. 571

112. 112.

When When symptoms symptoms to typho typhoid id fever fever first becom become e apparent, apparent, Salmon Salmonella ella typhi typhi is most most frequently isolated from: A. Feces B. Urine C. Blood D. Bone Bone marr marrow ow Blood - on the first week of infection, Feces - second week. Urine - third week. Reference: Zinsser p. 563

113. 113.

The area area in the body body where where Salmo Salmonell nella a typhi typhi resides resides if one one is a chronic chronic carrier  carrier  A. smal smalll intes intesti tine ne B. kidney C. bloo blood d str strea eam m D. gall gall bla bladd dder  er  Gall bladder - where organisms are found and are excreted in the stool and urine of the chronic carrier of Salmonella. Reference: Zinsser p. 562

114. 114.

A 3 year old old boy was diagn diagnose ose to have have menin meningiti gitis s CSF cultur culture e on Chocola Chocolate te agar  agar  showed growth of small gram negative rods. The most probable etiologic agent is: A. N. men menin ingi giti tide des s B. S. aureus C. Group Group B Strep Streptoco tococcu ccus s D. H. inf influ luenz enzae ae type type b H. influenzae type b - is the most commonly isolated etiologic gram negative rods causing meningitis. N. meningtidis - gram negative plo. Group B Strep gram positive cocci. Reference: Zinsser p. 462

18

115. 115.

Among Among the gram gram negative negative rods rods producin producing g gastroent gastroenterit eritis, is, the most most infectio infectious us is: A. V. chol choler era a B. E. coli coli (015 (0157) 7) C. Sal. Sal. Typ Typhim himuri urium um D. Shig. Shig. Dysen Dysenteri teriae ae type type 1 Shigella - only 200 cells is enough to cause diarrhea while A-C you need to 10 3 - 10 5 cells to produce infection. Reference: Tortora et al

116. 116.

Pneu Pneumo mococ cocci ci prim primar arily ily caus cause e diseas disease e by: A. toxin toxin produc producti tion on B. enzy enzyme me produ product ction ion C. eluding eluding phagocytosi phagocytosis s and favoring favoring invasiveness invasiveness D. produci producing ng delayed delayed immunolog immunologic ic reaction reaction Pneumococci does not produce toxins and enzymes. Capsules are virulence factors which interfere with phagocytosis and favoring invasiveness. Reference: Zinsser p. 435

117. 117.

The formatio formation n of pseudom pseudomemb embrane rane in dipht diphther heria ia is due to: A. inhibitions inhibitions by toxin toxin of protein protein synthesis synthesis by ADP ribosyslatio ribosyslation n of EF-2 B. spores spores which which spread spread subcut subcutane aneousl ously y C. invasion of mononuclear mononuclear phagocytic phagocytic cells D. neutralization neutralization of exotoxin exotoxin by antibody antibody of blocking blocking the interaction interaction of fragment fragment B with receptors Diphtheria toxin inhibits protein synthesis of eukaryotic cells thereby causing cell death. Pseudomembrane formation is due to accumulation of necrotic tissue. Reference: Zinsser p. 488

118.

Lyme disease is an an endemic endemic inflammatory inflammatory disorder disorder with this distinctive distinctive skin skin disorder. disorder. A. Chron hronic ic der derma mati titi tis s B. eryt erythe hema ma marg margin inat atum um C. eryth erythema ema chron chronic icum um migra migrans ns D. ecth ecthym yma a gang gangre reno nosu sum m Erythema chronicum migrans is a tick-borne associated syndrome. Reference: Lippincott Illustrated Review in Microbiology p. 225

119.

Serology Serology in the diagnosis diagnosis of Syphilis Syphilis using a non-trepon non-treponemal emal tests is the the following following EXCEPT: A. Microhem Microhemaggl agglutin utinatio ation n assay assay B. Venereal Venereal Disease Research Research Laboratory Laboratory C. Fluores Fluorescent cent Antibod Antibody y Tests Tests D. Reiter Reiter Protein Protein Comple Complement ment Fixatio Fixation n Two different types of test are used. Non-treponemal tests detects Wasserman or reagenic antibodies; a few examples of the nontreponemal tests include VDRL, VDRL, automated regin, Kahn, plasmacrit, Hinton and Kline tests. Treponemal tests detect antibodies specific for treponemal antigens; they include FTA=ABS, T. pallidium immunobolization and hemagglutination tests. Reference: Zinsser p. 663-664

19

120. 120.

Mycoplas Mycoplasmas mas are are very pleomo pleomorphi rphic c and cannot cannot be classif classified ied as to shape shape because because they they lack the following: A. Peptidoglycan B. lipid bilayer  C. nucleu leus D. lipop lipopoly olysa sacc cchar haride ide Lacking cell walls, mycoplasmas are enclosed instead by a membrane composed of a lipid bilayer. They are therefore plastic and very pleomorphic. Reference: Zinsser p. 733

121. 121.

A definitiv definitive e identifica identification tion of M. tubercu tuberculosi losis s can be obtaine obtained d by doing doing the followin following g procedure? A. ZiehlZiehl-Ne Neels elsen en sta stain in B. AFB AFB Cul Cultu ture re C. Biopsy Biopsy of Tube Tubercul rculin in test test D. Nucleic Nucleic acid acid amplif amplificat ication ion Cultures can detect small numbers of organism in the original sample. The presence of Acid-fast organism on Ziehl Neelsen cannot distinguish on morphologic morphologic groups M. tuberculosis from the other mycobacteria mycobacteria that are saprophytic. Nucleic acid amplification utilizes enzymes that rapidly IGS ribosomal RNA which can be used for patients with positive smears and while culture results are pending. Reference: Lippincott Illustrated Review for Microbiology p. 251

122. 122.

Rifamfic Rifamficin in a broad antim antimicro icrobia biall agent is effecti effective ve treatmen treatmentt for tubercu tuberculosis losis by: by: A. selective inhibition of the biosynthes biosynthesis is of the arabinoga arabinogalactan lactan and and lipoarabinomannan B. mediati mediating ng drug perm permeabi eability lity and and efflux efflux C. Inhibiting Inhibiting peptidoglycan peptidoglycan synthesis synthesis by blocking action of D-alanise D-alanise synthesis D. binding to RNA polymerase polymerase thereby thereby interfering interfering with with mRNA synthesis synthesis Rifamficin is a broad spectrum antimicrobial agent that acts the interferring with the synthesis of mRNA by binding to RNA polymerase. Selective inhibitions by ethamentol of the biosynthesis of arabinogalactan and lipoarabinomannan, key structural components of the mycobacterial cell wall. Cyclosine inhibits synthesis of peptidoglycan by blocking the action of the Dalanine synthetase. Reference: Tortora p. 561 123. 123.

In prima primary ry disea disease, se, M. M. tubercu tuberculosi losis s surviv survives es and and grows grows within within these these host host cells cells A. macr macrop opha hage ges s B. basophils C. neut neutro roph phil ils s D. eosi eosino noph phil ils s M. tuberculosis may continue to survive and remain dormant in activated macrophages where it is protected from phagocytosis Reference: Lippincott Illustrated Review of Microbiology p. 247

124. 124.

Members Members of the the genus genus Ricketts Rickettsia ia are are morpho morphologi logicall cally y & bioch biochemic emically ally similar similar to to A. gram gram-po -posit sitive ive bacte bacteria ria B. gram negative bacteria C. clos clostr trid idia ia D. virus ruses

20

Rickettsias are short-rod shaped or cocci bacillary organisms measuring 0.8 to 2.0 um long Reference: Lippincott Illustrated Review of Microbiology p. 259 125. 125.

The hallm hallmar ark k of vira virall infec infectio tion n of the the cell cell is is the the prod product uction ion of  of  A. inclus inclusion ion bodie bodies s B. multinu multinuclea cleated ted giant giant cells cells C. cyto cytopa pathi thic c effec effectt D. granu granule le form format ation ion The hallmark of viral infection of the cell is the cytopathic effect. This change begin with a rounding and darkening of the cell and culminates in either lysis or giant cell formation. Infected cells cells frequently contain inclusion bodies which are discrete areas containing viral particles. Reference: Zinsser p. 943

126. 126.

The comple complete te infec infectio tious us form forms s of of Hep Hepati atitis tis B is: is: A. HBeAg B. Dane Dane parti particl cles es C. HBsAg D. HBcAg The Dane particle is the complete form of Hepatitis B whereas the antigens are subunit forms of the surface and core of the virus. Reference: Zinsser p. 1040

127. 127.

Which Which of of the the follow following ing virus/es virus/es contain/ contain/s s Hemagg Hemagglutin lutinin in on on its its surfac surface? e? A. Hepat epatit itis is B B. Hepat epatit itis is C C. Influ Influen enza za virus virus D. all of the the abov above e The antigen hemagglutinin is characteristic of Influenza virus. Reference: Zinsser p. 995

128. 128.

In their their multi multiplic plicatio ation, n, they they produc produce e DNA which which is integr integrate ated d into into the cell cell genom genome. e. A. Hepa Hepadn dnav avir irus us B. Reovirus C. Pico Picorn rnav avir irus us D. Herp Herpes esvi viru rus s Hepadnavirus transforms non-permissive cells by integration of DNA transcripts into host cell genome. They carry an RNA-dependent DNA polymerase (reverse transcriptase). Reference: Zinsser p. 810

129. 129.

The most most com commo mon n intra intraut uteri erine ne vira virall infec infectio tion n is cause caused d by: by: A. Cyto Cytomeg megalo alovir virus us B. Rubella C. Aden Adenov ovir irus us D. Herpe Herpes s simple simplex x Cytomegalovirus accounts for the biggest # of intrauterine infection followed by rubella. Reference: Zinsser p. 962

21

130. 130.

Smallpo Smallpox x was was eradic eradicated ated by the global global use use of of vaccin vaccine e which which contain contains: s: A. kill killed ed viru virus s B. attenua attenuated ted live virus virus C. prefo preform rmed ed antibod antibodies ies D. preformed preformed antibodies antibodies and live live attenuated attenuated virus Smallpox disease was eradicated by use of a live attenuated vaccine. Live vaccine are used because duration of immunity is longer with greater  effectiveness of protection, both IgA and IgG are elicited when administered by the natural route of infection which is intradermally where the virus replicates. Cell mediated immunity is produced. Reference: Zinsser p. 949

131. 131. The most most importa important nt labora laboratory tory test for the the detec detection tion of early early Hepatit Hepatitis is B virus virus infection is: A. immunoa immunoassay ssay for HBsAg HBsAg B. immunoassay for HBeAg C. immuno immunoassa assay y for for Anti Anti HBc HBc D. immuno immunoassa assay y for for Anti Anti HBs HBs HBsAg appears during the incubation period and is detectable in most patients during the prodrome and acute phase; falls to undetectable levels during convalescence. HBeAg arises during the incubation period and is present during the prodome and early acute disease. Its presence is an important indicator of transmissibility. Anti HBc is detectable during prodrome and acute disease. Anti HBs is detectable during late convalescence. Reference: Zinsser p. 1043 132. 132.

A commo common n upper upper respirat respiratory ory tract tract disease disease produce produced d by adenovir adenovirus us is: is: A. Rhin hiniti itis B. atypi atypical cal pneum pneumon onia ia C. pharyng pharyngoco oconjun njunctiv ctivitis itis D. paro paroti titi tis s Adenoviruses cause a variety of upper and lower respiratory tract disease. Pharyngoconjunctivitis is common. Reference: Zinsser p. 970

133. 133. The virus virus that that is increasi increasingly ngly being being recogn recognized ized as cause cause of nonbac nonbacteri terial al infant infant diarrhea A. Retro etrovi viru rus s B. Rota Rotavi viru rus s C. Orbi Orbivi viru rus s D. Cali Calici civi viru rus s Rotavirus gastroenteritis is one of the most common childhood illness and a leading cause of infant deaths in developing countries. Reference: Zinsser p. 993 134. In the laboratory laboratory this protein protein from Influenza virus virus agglutinates agglutinates red blood blood cells whereas whereas in vivo, it binds to the surface receptor of the host cell to initiate infection. A. Hemag Hemaggl glut utin inin in B. Neur Neuram amid idas ase e C. Hemol emolys ysin in D. caps capsid id anti antige gen n The Influenza virus has an envelope covered with 2 different spikes Hemagglutinatinin and neuramidase. Hemagglutination initiates infection in 22

the host and in the lab causes agglutination of RBC's. Neuramidase cleaves neuramic acid (sialic acid) to release progeny virus from the infected cell. Hemolysin is from bacterial cells and causes hemolysis of RBC. Reference: Zinsser p. 778 135. 135.

In the the preven prevention tion of poliom poliomyele yeletis, tis, the the use use of oral oral vacc vaccine ine is preferre preferred d becaus because: e: A. induce induces s intes intestin tinal al IgA B. induces humoral IgG C. can produce lifelong immunity D. can produce produce immunity to unimmunized unimmunized adults The oral vaccine or Sabin vaccine which is a live attenuated vaccine is preferred over the Salk vaccine or killed vaccine because (1) it interrupts fecal-oral transmission by inducing secretory IgA in the GIT; killed vaccine does not. Both kinds of vaccine induce humoral IgG. Infection provides lifelong type specific immunity. Reference: Zinsser p. 981

136.

Hand-foot-and-mouth disease characterized by versicular rash on the hands and feet and ulcerations in the mouth, mainly in children is caused by: A. Echo Echovi viru rus s B. Para Parainf influe luenz nza a virus virus C. Coxsack Coxsackie ie virus virus Group Group A D. Coxsack Coxsackie ie virus virus Group Group B Echovirus causes aseptic meningitis. Parainfluenza causes croup. Coxsackie B causes pleurodynia. Coxsackie A causes herpangina and hand-foot-andmouth disease. Reference: Zinsser p. 774 137. 137.

The mos mostt commo common n funga fungall opport opportun unist istic ic infec infectio tion n is caus caused ed by: by: A. Aspe Aspergi rgillu llus s fumiga fumigatus tus B. Cryptoc Cryptococcu occus s neoform neoformans ans C. Cand Candid ida a albi albican cans s D. Mucor  Candida albicans - it is endogenous, a lot of risk factors like pregnancy, diabetic person, prolonged used of antibiotic, used of birth control pills Reference: Zinsser p. 1139

138. 138.

The most most com commo mon n mode mode of infe infecti ction on of of system systemic ic fung fungii is thru thru:: A. inge ingest stio ion n B. traumatic implantation C. inha inhala lati tion on D. all of the the abov above e Inhalation of infectious particles (spores) which are suspended in the air. Reference: Zinsser p. 1091

139. A 45 year old post kidney transplant transplant patient develop fever and pneumonia pneumonia with marked neutropenia. Bronchial washing show dichotomously branching filamentous fungi. The fungi involved is: A. spro sproro roth thri rix x B. cryp crypto toco cocc ccus us C. candid dida D. aspe asperg rgil illu lus s

23

In immunocompromised patient, inhalation of spores of Aspergillus can lead to pneumonia and are seen as dichotomous branching hyphae (filamentous). Reference: Zinsser p. 1127 140. In a 7 year year old malnourish malnourished ed boy with with multiple multiple nematode nematode infections, infections, the the most severe severe complication will come from: A. Ascaris Ascaris lumbrico lumbricoide ides s B. Trich Trichuri uris s trich trichiur iura a C. Neca Necator tor ameri america canus nus D. Enterob Enterobius ius vermicul vermicularis aris Ascaris has a tendency to migrate to other openings causing more severe disease. Reference: Brown 6 th ed. p. 141-142, Roberts and Janovy p. 422 141. 141.

Loeffler Loeffler’s ’s syndr syndrome ome can be seen seen in in the the follow following ing paras parasitic itic infecti infections ons EXCE EXCEPT: PT: A. Asca Ascari rias asis is B. Strongyloidiasis C. Necatoriasis D. Tric Trichu huri rias asis is Loeffler’s syndrome is eosinophilic lung. As the parasitic larvae (rhaditiform for Ascaris, filariform for Strongyloides and Necator) travels to the pulmonarycirculatory system, the body reacts with an anaphylactic type of  hypersensitivity that produces the eosinophilic lung condition. Trichuris does not have a pulmonary circulatory pathway. Reference: Brown 6 th ed. p. 141, 126, 130; Roberts & Janovy p. 386, 422

142. 142.

All All cesto cestodes des requir require e an interm intermedi ediate ate host host EXC EXCEP EPT: T: A. Diphy Diphyllo llobot bothri hrium um latum latum B. Taenia saginata C. Hymen Hymenol olep epis is nan nana a D. Echinoc Echinococcu occus s granulos granulosus us Man gets H. nana infection by ingesting the eggs directly. All other cestodes require an intermediate host Reference: Brown 6 th ed. p. 192

143. 143.

Cystice Cysticercos rcosis is due due to to Taenia Taenia solium solium is acqui acquired red through through ingestio ingestion n of: of: A. cysti cystice cerc rcus us larva larva B. cysticercoid larva C. proce procerco rcoid id larv larva a D. egg A more dangerous sequence can occurs when a person ingests the egg of  Taenia solium in contaminated food or water rather than eating raw or  undercooked pork containing the larva cystercus. Cysticercoid larva pertains to D. caninum and H. nana. Procercoid larva is for D. latum. The ingested egg hatch in the small intestines and the oncopheres burrow through the wall into a blood vessel, where they can disseminate to many organs. This condition is called cysticercosis. Reference: Brown 6 th ed. p. 207

24

144. 144.

The infect infective ive stage stage to to man man of Neca Necator tor ameri america canus nus is: A. rhabd rhabditi itifo form rm larva larva B. filariform larvae C. embry embryona onate ted d egg egg D. Micr Microf ofil ilar aria ia Filariform larvae of hookworms penetrate the skin Reference: Roberts & Janovy p. 410

145. 145. This cestode cestode possess possess a quadra quadrate te scole scolex x with with 4 circular circular suckers suckers and no roste rostellum llum or hooklets: A. Diphy Diphyllo llobot bothri hrium um latum latum B. Hyme Hymeno nolep lepis is nana nana C. Taenia saginata D. Echinoc Echinococcu occus s granulos granulosus us D. latum has an elongated scolex. H. nana has a globular scolex with 4 suckers, a short retractile rostellum armed with a single row of hooklets. E. granulosus has 4 suckers with a rostellum and a double row of hooklets. Reference: Brown 6 th p. 194, Roberts & Janovy p. 333 146. 146.

Amoe Amoebic bic absc abscess ess of the the live liverr is mostl mostly y foun found d in the the A. left left antero anterome medi dial al lobe lobe B. right posterosuperior lobe C. inferior lobe D. middle lobe Spread of amoeba from the large intestines is by the portal vein and deposit on the right posterosuperior lobe Reference: Chatterjee p. 25

147.

Little Joey was frequently sick of various infectious diseases. He was found to have low CD4+ lymphocytes titers. Which of the following immune response processes would still be functional in Joey’s body? A. T cell cell help help for for B lymphoc lymphocytes ytes B. Recogn Recognition ition of of peptidepeptide-MHC MHC compl complex ex C. Phagocytic Phagocytic function function for for foreign foreign molecules molecules D. T-depend T-dependent ent antigen antigen recog recogniti nition on This is an innate natural cellular immune ability of the host to respond to foreign molecules which is independent of the Th series. Reference: Stanley p. 92; 124 148. 148.

The host host immu immune ne resp respon onse se in viral viral infec infectio tions ns is most mostly ly by: by: A. Th ce cells B. B cells C. Tc ce cells D. NK ce cells lls Viral infections produce endogenous peptides which are presented on the surface of cells in conjunction with MHC Class 2. These are recognize primarily by Tc cells. B cells are mostly for exogenous peptides that are antibody mediated mainly stimulated by Th cells while the NK cells’ major  activity is to directly destroy aberrant cells such as in cancer. Reference: Roitt, Brostoff & Male p. 15.3-15.4

25

149. 149. A killed killed virus virus is the the major major compone component nt of the vaccine vaccine against against this this patho pathogen genic ic organism. A. measle asles s B. rubella C. mumps D. rabies Measles, rubella, mumps vaccines contain living but attenuated virus (also for  chickenpox, Sabin polio) Rabies vaccine (also for Hep A, Salk polio, influenza) contain killed or inactivated viruses. Hep B vaccine contains antigenic fragments of the virus usually using recombinant technology (yeastderived). Subunit vaccines are also used as in pertussis. Tetanus and diphtheria utilize toxoids which are inactivated toxins or antitoxins, antibodies against these. H. influenzae type b vaccine is conjugated to a protein which can be a diphtheria toxoid. Reference: Tortora p. 502 150. 150.

Alkaline Alkaline pH in urine urine seen seen in Proteus Proteus is due due to its ability ability to split split urea into: into: A. NH3 & C02 B. Cl & Ca C. Ca & Mg D. Na & Cl Answer: A (pp.208)

151. 151.

It is is respo responsib nsible le for Escheri Escherichia chia coli causing causing meningi meningitis tis in infants infants A. H antigen B. LPS C. K ant antig igen en D. O anti antige gen n Answer: C (pp.208)

152. 152.

Vi antig antigen en for for Salm Salmone onella lla typhi typhi A. penetrates epithelial lining of the intestinal tract B. antigenic antigenic structure structure associated associated with invasiven invasiveness ess C. interfer interfere e with antibody antibody bindin binding g to bacteria bacteria D. confers a certain degree of immunity Answer: B (p 209)

153.

Initial recognition event for the activation of the alternative pathway A. C3b B. C1 C. C4 D. Fact actor D Answer: A (p 108)

154.

The characteristic morphologic feature of influenza responsible for antigenic changes of  the virus A. Envelope B. segme segmente nted d geno genome me C. capsid D. nucl nuclei eic c aci acid d Answer: B (p 320)

26

155.

Replicate in the cytoplasm of the host cell A. Aden Adenov ovir irus us B. Ortho Orthomy myxov xoviru irus s C. Poxv Poxvir irus uses es D. HIV Answer: C (p332)

156.

The exudative lung parenchymal lesion with lympadenopathy in primary tuberculosis infection is called A. tube tuberc rcul ulos osis is B. gran granul ulom oma a C. ghon ghon com compl plex ex D. fibro fibrotic tic lesion lesion Answer: C (p278)

157.

Which of the following statement/s is correct regarding neonatal herpes A. infections are almost always asymptomatic B. most common route of infection is thru contact with herpetic lesion during vaginal delivery C. caus causes es late latent nt infe infect ctio ion n D. usua usuall lly y cau cause se by HSV HSV 1 Answer: B (p376)

158. 158.

An 18 year year old colleg college e student student develo developed ped fever fever follow followed ed by jaundi jaundice. ce. Acute Acute viral viral hepatitis was entertained. Results of the hepatitis profile are as follows: anti-HAV IgM (-), anti-HbsAg (-), anti-HBc IgM (+), anti-HBc IgG negative. He most probably has: A. Hepatitis A B. Hepatitis B C. Hepatitis C D. Hepatitis E Answer: B ( p 410)

159.

A 21 y/o male with history of sexual contact 4 days ago resents with painful urination and purulent penile discharge. Which of the following is most likely? A. prim primar ary y syph syphil ilis is B. genit nital her herp pes C. gonorrhea D. lymp lympho hogr gran anul ulom oma a vener venerum um Answer: C (p259)

160.

A 1 and half y/o boy developed fever, coryza and cough followed 4 days later by maculopapular rashes starting from the hairline and becoming generalized. Fever  persisted. The most likely etiology is a/an A. RNA virus B. DNA virus C. gram gram posi positi tive ve cocc coccus us D. gram gram nega negati tive ve baci bacill llus us Answer: A ( p 470)

27

161.

A neonate is born with the following features: microcephaly, jaundice, poor suck and chorioretinitis. A congenital infection is entertained. The most common congenital infection is due to: A. Chlam hlamy ydia dia B. HIV C. CMV D. HSV Answer: C ( p 382)

162.

A 2 month old girl was brought to you because of milk curd-like lesions in the mouth noted for a week. The whitish lesions were adherent to the oral mucosa with pinpoint bleeding when the mother tried to remove them. This is caused by A. herp herpes es sim simpl plex ex 1 B. Coxsa oxsack ckie ie A C. alb albican icans s D. neof neofor orma mans ns Answer: C ( p 550)

163.

A 23 y/o office worker found out that his sexual partner had HIV infection. The screening procedure of choice for HIV infection is A. PCR B. HIV Elisa C. western blot D. nort orther hern blot lot Answer: B (p 525)

164.

A 6 month old baby has acute watery diarrhea. Which of the following is the most common cause of diarrhea in this age group? A. ETEC B. Rotavirus C. EPEC D. V. cholerae Answer: B ( p434)

165.

A health worker who did not receive prior mumps vaccine developed parotid swelling and fever. The following belongs to the same virus family as mumps virus A. A. germ german an meas measles les B. RSV C. Infl Influe uenz nza a D. Japan Japanes ese e B virus virus Answer: B ( p 471)

166.

A 25 yr old saleslady developed high fever, chills and cough. On PE she had crackling rales on the right lung base. The diagnosis is pneumonia. The most common bacterial cause of this condition in this age group is: A. S. au aureus reus B. N. men menin ingi giti tidi dis s C. S. pneumoniae D. Hemophi Hemophilus lus influenz influenzae ae Answer: C ( p 213)

28

167.

Infection with this organism is the most common risk factor for cervical CA A. HIV B. N. gono gonorr rrhe hea a C. HPV D. HSV 2 Answer: C ( p512)

168.

Most common cause of UTI in women is characterized by A. Urea Urease se prod produc ucti tion on B. frui fruity ty odo odorr on cul cultu ture re C. gree greenis nish h met metall allic ic shee sheen n D. swar swarmi ming ng phen phenom omen enon on Answer: C ( p 219)

169.

Which of the following laboratory tests can be used to monitor response to treatment in syphilis? A. FTA-ABS B. VDRL C. Dark Darkfi fiel eld d micro microsc scop opy y D. ELISA Answer: B ( p 287)

170.

An encapsulated fungi that causes meningitis is A. H. capsu psulat latum B. neoformans C. C albicans D. immitis Answer: B ( p 552)

171. 171.

The follo following wing statemen statements ts are are true true regardi regarding ng N. meningit meningitidis idis A. it is the only specie that is oxidase positive B. oxid oxidize izes s mal malto tose se and and glu gluco cose se C. oxid oxidiz izes es gluc glucos ose e only only D. it is is the the only only gra gram m nega negativ tive e speci specie e Answer: B (p256)

172.

The following statements are true about S. aureus A. it is cat catala alase se neg negati ative ve B. can secrete preformed toxins C. prefers prefers anaer anaerobic obic conditi condition on D. cannot grow in 7.5% NACL Answer: B (p.199)

173.

Toxic shock syndrome secondary to Staphylococcus tend to commonly cause infection among A. tamp tampon on user users s B. asth astham amti tics cs C. IV drug drug user users s D. inf infant ants Answer: A (p 201)

29

174.

A 14 year old male has high fever for a week, tonsillitis, hepatosplenomegaly and cervical lymphadenopathy. Peripheral smear shows 10% atypical lymphocytes. The most likely diagnosis is: A. Streptoc Streptococca occall sore sore throat throat B. leu leukemia emia C. infectio infectious us monon mononucle ucleosis osis D. diph diphth ther eria ia Answer: C (p 386)

175.

Streptococcal sore throat is definitively diagnosed by A. ASO tititer  ter  B. throa throatt swab swab cultu culture re C. rapid rapid antigen antigen testing testing from from throat throat swab D. Gram Gram sta stain in Answer: B (p 210)

176.

Which of the following, when positive, indicates an acute Epstein-Barr virus infection? A. anti anti-E -EBN BNA A B. anti anti-V -VCA CA IgG IgG C. anti anti-V -VCA CA IgM IgM D. anti anti-E -EBV BV DNA DNA Answer: C (p 388)

177.

The hallmark of Dengue Hemorrhagic Fever is A. throm thrombo bocyt cytope openia nia B. shock C. plas plasma ma loss loss D. positive positive tourniqu tourniquet et test Answer: C (p 452)

178.

Dengue virus has how many serotypes? A. one B. two C. three D. four  Answer: D ( p 452)

179.

The most common vector of Dengue virus is ( p 452) A. Anop Anophe heles les minimu minimus s B. Mans Manson onia ia sp. sp. C. Aede Aedes s aegy aegypt ptii D. Aede Aedes s albo albopic pictu tus s

180.

A 1 year old girl has high fever for four days, coryza, hacking cough and poor  appetite. On PE, she has congested pharynx and Koplik’s spots. The causative agent of  this condition A. is a DNA DNA viru virus s B. has has thre three e sero serotyp types es C. belongs belongs to the the paramyxo paramyxoviru virus s family family D. has doubledouble-stra strande nded d RNA RNA Answer: C ( P482)

30

181.

Which of the following diseases is currently vaccine-preventable? A. cytom cytomeg egal alic ic inclus inclusion ion disea disease se B. German man me measl asles C. Dengue fever  D. Hepatitis C Answer: B ( p 485)

182.

A 20 year old male developed fever, vomiting and poor appetite for four days followed by jaundice. A diagnosis of viral hepatitis is entertained. The results of the hepatitis profile are as follows: positive anti-HAV IgM, negative antiHBsAg, positive HBsAg, negative anti-HBc IgM. Which is correct? The patient has acute Hepatitis A and is most likely a Hepatitis B carrier  B. He has has acute acute Hepat Hepatiti itis sB C. He has has acute acute Hepatit Hepatitis is C D. He does does not have have acute acute viral hepat hepatitis itis A.

Answer: A ( p 411) 183.

The protective antibody following immunization against Hepatitis B is A. anti anti-H -HBc Bc IgM IgM B. anti anti-H -HBc Bc IgG IgG C. anti anti-H -HBs BsAg Ag D. anti anti-H -HCV CV Answer: C ( p 415)

184.

Which of the following agents of viral hepatitis is fecal-oral in transmission and causes more severe disease in pregnant women? A. Hepat epatit itis is A B. Hepatitis B C. Hepa Hepati titi tis sC D. Hepa Hepati titi tis sE Answer: D ( p 408)

185.

Bovine spongiform encephalopathy (mad cow disease) is caused by an agent which has the following characteristic A. doub doublele-st stran rande ded d DNA B. singl single-s e-stra trande nded d RNA RNA C. doub doublele-st stran rande ded d RNA RNA D. does does not not contain contain nucleic nucleic acid acid Answer: D ( p 500)

Human influenza virus which undergoes antigenic shift and can cause pandemics influenza A influenza B influenza C highly-pathogenic avian flflu 186.

. . . .

Answer: A (p 463)

31

187.

Fetal hydrops is a prominent complication of maternal infection with A. rubella B. parvovirus B19 C. measles D. cytomegalovirus Answer: B ( p 357)

188. 188.

An 8 month month old old girl girl develope developed d high high fever fever for three three days days but but was other otherwise wise playf playful. ul. On the fourth day, fever was gone but she had maculopapular rashes on the face which rapidly spread all over the body. This common disease is caused by A. B. C. D.

measles les viru irus huma human n he herpes rpesvi viru rus s6 parvo rvoviru irus B1 B19 rube rubellla virus irus Answer: B ( p389)

189. 189.

A 22 year year old sex care care worker worker wante wanted d to be tested tested for for human human immunode immunodefici ficienc ency y virus (HIV). What test would you initially recommend? A. B. C. D.

Western bl blot HIV Elisa CD4:CD8 CD8 ratio tio VDRL Answer: B ( p 526)

190.

HIV infection causes the ratio of CD4 to CD8 T cell ratio to fall. CD4 T cells are also called A. T cyto cytoto toxi xic c cell cells s B. cyto cytoly lyti tic c T cell cells s C. T helpe lper cell ells D. supp suppre ress ssor or T cel cells ls Answer: C (p 521)

191.

The major function of CD8 T lymphocytes is A. to kill kill cells cells infected infected by intra intracel cellul lular ar pathoge pathogens ns like like viruses viruses B. antibody antibody-de -depen pendent dent cellular cellular cytotoxi cytotoxicity city C. opsoniz oniza ation ion D. phago agocyt cytosis Answer: A (p 116)

192.

A one and a half year old boy has recurrent bacterial pneumonia and middle ear  infection. An antibody deficiency is entertained. Antibodies are produced in large quantities by A. B lymp ymphocy ocytes B. T lym lymphocyt cytes C. plasma ce cells D. mast cells Answer: C (p 125)

32

193.

Immunoglobulins aid phagocytosis because they can bind to phagocytes via A. Fab rec recept eptors ors B. Fc re receptors C. CD4 rec receptors tors D. C3b rece ecepto ptors Answer: B ( p 125)

194.

The following virus/viruses exhibit/s latency in nerve ganglia A. smallp llpox viru irus B. chic chicke kenp npo ox viru virus s C. cyto cytome mega galo lov virus irus D. adenovirus Answer: B (p 378)

195.

A virus which infects B lymphocytes by attaching to the latter’s complement Receptors A. cyto cytome mega galo lov virus irus B. Epst Epstei einn-Ba Barr rr vir virus us C. Japanese B encephalitis virus D. HIV Answer: B ( p 386)

196.

A 1 year old infant had exposure to a caregiver with cavitary tuberculosis. Chest x-ray was negative. Mantoux test done showed induration of 12 mm after 2 days. The Mantoux tuberculin test result is an example of  A. Type Type I hyp hyper erse sens nsit itiv ivit ity y B. Type Type II II hype hypers rsen ensi siti tivi vity ty C. delay delayed ed-ty -type pe hyp hypers ersen ensit sitivi ivity ty D. Arth rthus re react action ion Answer: C ( p 130)

197. 197.

The followi following ng are are char characte acterist ristics ics of M. M. tuberc tuberculos ulosis is A. have have minim minimal al lipid lipid content content in their their cell wall B. canno cannott be gro grown wn on on artif artifici icial al medi media a C. retain retain carbol carbolfuch fuchsin sin dye dye after after exposu exposure re to acid-a acid-alco lcohol hol D. appea appearr as rou round nd cha chains ins on Gram Gram stain stain Answer: C ( p25)

198.

Subacute sclerosing panencephalitis is a late complication of infection with this virus A. Varicella B. Rubeola C. West Ni Nile virus irus D. poliovirus Answer: B ( p483)

33

199. 199.

A newborn newborn presents presents with with lethargy lethargy,, purpura, purpura, choriore chorioretin tinitis itis and pulmon pulmonary ary infiltra infiltrates. tes. Which of the following could be a cause of this syndrome? A. Amebiasis B. Candidiasis C. Cyto Cytome mega galo lovi viru rus s D. Huma Human n papi papill llom omav avir irus us ANS: C Medical Microbiology by Jawetz.,23rd edition, page 445

200.

Which one of the following is a subcellular subcellular structure structure that is found only in Gram Gram negative negative bacteria? A. endospores B. lipo lipopo poly lysa sacc ccha hari ride de C. mitoc itoch hond ondria D. phosp phospha hatid tidyl yleth ethano anolam lamine ine ANS: B Medical Microbiology by Jawetz., 23rd ed., page 28

201.

Which one of the following statements statements best best describes describes a function function of the peptidoglycan peptidoglycan layer in bacteria? A. The abilit ability y of bacteri bacteria a to survive survive chan changes ges in the the osmolari osmolarity ty of their  their  environment is due to the physical properties of the peptidoglycan layer. B. The peptid peptidogly oglycan can layer layer contains contains all all the enzymes enzymes respo responsib nsible le for the synthesis of membrane phospholipids. C. The peptid peptidogly oglycan can layer layer protect protects s bacteria bacteria from from the effects effects of ultravi ultraviole olett light. D. The peptid peptidolgy olgycan can layer layer renders renders Gram Gram negative negative bacter bacteria ia resistan resistantt to detergents. ANS: A Medical Microbiology by Jawetz., 23rd ed., page 22

202.

4. Which Which one one of the following following structures structures is responsible responsible for the passage passage of low molecular  molecular  weight solutes through the outer membrane of Gram negative bacteria? A. capsules B. endospores C. flagella D. porins ANS: D Medical Microbiology by Jawetz., 23rd ed., page 27

203.

Your patient, suffering from a bacterial bacterial infection, infection, receives receives a penicillin penicillin injection and almost immediately experiences respiratory distress and loses consciousness. This reaction is MOST LIKELY mediated by A. comp comple leme ment nt.. B. CD8+ CD8+ cytol cytolyti ytic c T cells cells.. C. macr macrop opha hage ges. s. D. mast mast cell cells. s. ANS: D Medical Microbiology by Jawetz., 23rd ed., page 140

34

204. 204.

Which Which one of thes these e statemen statements ts explain explains s the origin origin of Rh Rh antibodie antibodies s in the blood blood of an Rh-negative mother? A. They are are naturally naturally occurring occurring in the blood of all Rh-negative Rh-negative persons. persons. B. They are are received received through through mis-match mis-matched ed blood transfusions. transfusions. C. They are are synthesized synthesized by the mother mother in response response to Rh-positiv Rh-positive e cells from her first child. D. They are transferre transferred d from the Rh-positive Rh-positive father father during during intercourse. intercourse. ANS: C Medical Microbiology by Jawetz., 23rd ed., page 135

205.

Which one of the following bacterial pathogens requires selective culture conditions for  reliable isolation from the stool, including media with antibiotics and incubation at 42 ° C in increased CO2? A. Camp Campyl ylob obac acte terr jeju jejuni ni B. Enter Enterohe ohemo morr rrhag hagic ic Esche Escheric richi hia a coli C. Salmon lmonel ella la sp. D. Shigella sp. ANS: A Medical Microbiology by Jawetz., 23rd ed., page 273

206. 206.

11. Which Which one of the the followin following g is considere considered d facultati facultatively vely intrac intracellu ellular lar for human human macrophages? A. Enter Enteroin oinva vasiv sive e Esch Escher erich ichia ia col colii B. Enter Enteroto otoxig xigen enic ic Esche Escheric richia hia col colii C. Salm Salmon onel ella la typh yphi D. Shigella sp. ANS: C Medical Microbiology by Jawetz., 23rd ed., page 258

207.

A 20-year-old 20-year-old female presents with severe severe pelvic pain, fever, fever, nausea and vomiting. vomiting. She was on the second day of her menstrual cycle. A purulent cervical discharge was detected upon physical examination. Oxidase positive, Gram-negative diplococci were isolated on Thayer Martin agar from the endocervical swab. What is the MOST LIKELY identity of this organism? A. Bord Bordete etella lla pert pertuss ussis is B. Chlam Chlamyd ydia ia trachom trachomat atis is C. Haem Haemop ophil hilus us ducreyi ducreyi D. Neisse Neisseria ria gonorrh gonorrhoea oeae e ANS: D Medical Microbiology by Jawetz., 23rd ed., page 295

208. 208.

A large large outbrea outbreak k of food poiso poisonin ning g occurred occurred at at a company company picnic picnic.. Eighty Eighty people people reported explosive vomiting and diarrhea within 3 hours after eating. All of the victims recovered within 24 hours without medical intervention. No known bacterial or viral pathogens were isolated from the stool of affected individuals. A survey of the food eaten at the picnic suggested that potato salad was the common source of the poisoning. The individual who prepared the salad had a sty on her left eye. Which one of the following is the MOST LIKELY cause of this outbreak? A. Clostri Clostridium dium perfring perfringens ens B. Salmo Salmone nella lla typhimu typhimuriu rium m C. Staphy Staphyloco lococcu ccus s aureus aureus D. Staphyl Staphylococ ococcus cus epide epidermid rmidis is

35

ANS: C Medical Microbiology by Jawetz., 23rd ed., page 223 209.

Gram-positive Gram-positive cocci and polymorphonucle polymorphonuclear ar leukocytes leukocytes are observed on Gram Gram stain of  sputum from a 60-year old alcoholic with pneumonia. Culture of the sputum reveals alpha-hemolytic, catalase-negative bacteria. Which one of the following laboratory tests is the MOST APPROPRIATE next step in the definitive identification of this organism? A. bacitra bacitracin cin sensiti sensitivity vity test B. beta-la beta-lactam ctamase ase produ productio ction n test test C. ELEK ELEK test test D. bile bile solu solubi bili lity ty test test ANS: D Medical Microbiology by Jawetz., 23rd ed., page 236

210. 210.

A 19-year 19-year old femal female e is admitted admitted to an outpati outpatient ent clinic clinic with with suprapub suprapubic ic pain, pain, dysuria dysuria and an urgency to urinate frequently. She is otherwise healthy. A clean catch midstream urine specimen was collected and sent to the laboratory. Numerous neutrophils are detected in the urine sample. The bacteriology laboratory reports that less than 102 Gram-positive cocci and greater than 105 Gram-negative bacilli were recovered per milliliter of urine. Which one of the following is the most likely cause of  her infection? A. Esch Escher eric ich hia coli coli B. Kleb Klebsi siel ella la pneu pneumo moni niae ae C. Pseu Pseudo domo mona nas s aeru aerugi gino nosa sa D. Stap Staphy hylo loco cocc ccus us aure aureus us ANS: A Medical Microbiology by Jawetz., 23rd ed., page 252

211. 211.

A 40-year 40-year old male male hog hog farmer farmer complai complains ns of interm intermitte ittent nt fever, fever, night night sweats sweats and and headaches to his physician. The patient is hospitalized and blood and bone marrow specimens are collected. The physician requests that the laboratory incubate the cultures for at least 6 weeks. Nine days after admission, Gram negative coccobacilli that require increased CO2 for growth are isolated from the blood and bone marrow cultures. The patient is most likely to be suffering from which one of the following diseases? A. Brucellosis B. Bubonic onic plag lague C. Legi Legion onna nair ire’ e’s s dis disea ease se D. Rock Rocky y Mou Mounta ntain in spot spotted ted fever  fever  ANS: A Medical Microbiology by Jawetz., 23rd ed., page 284

212.

Which one of the following diseases is most likely to respond favorably to treatment with a β -lactam antibiotic? A. Legi Legion onel ello losi sis s B. Myco Mycopl plas asmo mosi sis s C. Syph Syphil ilis is D. Trach rachom oma a ANS: C Medical Microbiology by Jawetz., 23rd ed., page 334

36

213. 213.

A 20-year 20-year old woman woman comes comes to your your clinic clinic complain complaining ing of an irritati irritating ng vaginal vaginal disch discharge arge.. Upon examination, you find that the cervix and vaginal walls walls appear normal. The discharge is thin and milky, pH 5.5, and has a fishy odor when treated with potassium hydroxide. You are unable to detect pseudohyphae, buds, or flagellates upon microscopic examination. A Gram stain reveals numerous curved Gram-negative rods, epithelial cells, and clue cells, cells, but relatively few Gram-positive Gram-positive rods or WBCs. WBCs. Cervical and vaginal cultures are negative for STDs. What is your diagnosis? A. Bacte Bacteria riall vagi vagino nosis sis B. Cervi ervici citi tis s C. Trich Trichom omon onias iasis is D. Vagin Vaginal al candidi candidias asis is ANS: A Medical Microbiology by Jawetz., 23rd ed., page 316

214. 214.

A 75-year 75-year old man man in a nursin nursing g home is treat treated ed with a broad broad spect spectrum rum antibi antibiotic otic for for bed sores. Two days later he develops bloody diarrhea. A stool specimen from the patient is positive in an ELISA test for a necrotizing cytotoxin, and his treatment is switched to vancomycin. The agent MOST LIKELY to be responsible for this clinical syndrome is A. Bacte Bactero roide ides s fragili fragilis. s. B. Camp Campylo ylobac bacter ter jejun jejuni. i. C. Clostri Clostridiu dium m difficil difficile. e. D. Escheri Escherichia chia coli O157:H7 O157:H7.. ANS: C Medical Microbiology by Jawetz., 23rd ed., page 209

215. 215.

Which Which one of the the followi following ng toxins toxins is respon responsibl sible e for scalde scalded d skin syndro syndrome? me? A. Alph Alpha a hemol hemolys ysin in B. Exfo Exfoli liat atin in C. Hyalu Hyaluro ronid nidas ase e D. Staphyl Staphylococ ococcal cal enter enterotox otoxin in ANS: B Medical Microbiology by Jawetz., 23rd ed., page 225

216. 216.

A 50-year 50-year old cancer cancer patien patientt with severe severe neutrop neutropenia enia becam became e septic septic two weeks weeks after  after  being admitted to the hospital. An oxidase-positive, Gram-negative rod that did not ferment glucose was isolated from the patient’s blood. Which one of the following organisms is the most likely cause of the patient’s sepsis? A. Enter Enteroba obacte cterr cloacae cloacae B. Esch Escheri erich chia ia coli coli C. Klebsie Klebsiella lla pneumo pneumoniae niae D. Pseudo Pseudomon monas as aeru aerugin ginosa osa ANS: D Medical Microbiology by Jawetz., 23rd ed., page 262

217.

Blood culture from a 12-year 12-year old with acute acute osteomye osteomyelitis litis yields yields Gram-pos Gram-positive itive cocci cocci that that are b hemolytic and catalase positive. Which one of the following laboratory tests is the MOST APPROPRIATE next step in the definitive identification of this organism? A. Coag Coagul ulas ase e test test B. Optoch Optochin in sensitiv sensitivity ity test C. Slide agglutinatio agglutination n test for capsular capsular antigen antigen type type D. Test Test for capacit capacity y to grow grow in bile esculi esculin n ANS: A Medical Microbiology by Jawetz., 23rd ed., page 225 37

218. 218.

Activat Activated ed macroph macrophages ages are are the effector effector cells cells in the expres expression sion of a protect protective, ive, cellcellmediated immune response against A. Coryne Corynebact bacteriu erium m diphth diphtheria eriae. e. B. Haemop Haemophilu hilus s influen influenzae zae type type b. b. C. Listeri Listeria a mono monocyto cytogen genes. es. D. Strepto Streptococ coccus cus pneum pneumonia oniae. e. ANS: C Medical Microbiology by Jawetz., 23rd ed., page 152

219.

A patient patient recovering recovering from a crushing crushing leg leg injury injury suffered suffered during during an an a motorcycle motorcycle accident accident develops a temperature of 100o F, a rapid pulse, and extreme pain at the site of his the wound two days after the accident. The skin overlying the patient’s sutured wound is brownish- white, shiny, and studded with vesicles. Gram stain of the watery exudate from the wound reveals Gram-positive rods, Gram-negative rods, and Gram-positive cocci. The Gram-positive Gram-positive rods are are MOST LIKELY A. Baci Bacillu llus s cereu cereus. s. B. Bacte Bactero roide ides s fragili fragilis. s. C. Clostri Clostridiu dium m perfri perfringe ngens. ns. D. Coryne Corynebact bacteriu erium m ulcera ulcerans. ns. ANS: C Medical Microbiology by Jawetz., 23rd ed., page 208

220. 220.

A 13-year 13-year old old and her her 10-yea 10-yearr old brothe brotherr are seen seen at an an emergen emergency cy room room with with complaints of double vision, difficulty swallowing, and progressive muscular weakness. A detailed history reveals that both children attended a church picnic three days earlier  but neither child has gastroenteritis. The MOST LIKELY presumptive diagnosis is botulism . A. botulism. B. Chin Chines ese e ric rice e syn syndr drom ome. e. C. Guil Guilla lann-Ba Barr rré é syndr syndrom ome. e. D. salmo lmonello llosis. is. ANS: A Medical Microbiology by Jawetz., 23rd ed., page 206

221.

Epidemics Epidemics of bacterial bacterial meningitis meningitis in young, young, previously previously healthy adults are MOST MOST LIKELY LIKELY caused by A. Esch Escheri erich chia ia coli coli K-1. K-1. B. Haemop Haemophilu hilus s influen influenzae zae type type b. b. C. Listeri Listeria a mono monocyto cytogene genes. s. D. Neisse Neisseria ria mening meningitid itidis is group group A. ANS: D Medical Microbiology by Jawetz., 23rd ed., page 303

222. 222.

A 55-year 55-year old woman woman develops develops endocar endocarditis ditis follow following ing oral oral surgery. surgery. Gram-pos Gram-positiv itive, e, alpha-hemolytic, catalase-negative cocci are isolated from a blood culture. The MOST LIKELY agent is A. Enterococcus Enterococcus faecalis (group D streptoco streptococci). cci). B. Staphyl Staphylococ ococcus cus aureus. aureus. C. Streptococcus Streptococcus pyogenes pyogenes (group A streptococc streptococci). i). D. Viridan Viridans s strepto streptococc cocci. i. ANS: D Medical Microbiology by Jawetz., 23rd ed., page 236

38

223.

A 2-month 2-month old old infant infant is admitted to the hospital with fever, fever, lymphocy lymphocytosis, tosis, and bouts bouts of  of  violent coughing that often end in vomiting. The infant’s mother reveals that the child has not as yet received any immunizations. The attending physician swabs the infant’s nasopharynx and requests that the swab be plated on routine media as well as a special medium (Bordet-Gengou) that contains blood and glycerol. No organisms are found on blood or chocolate agar, but small Gram-negative rods are isolated on BordetGengou agar. The MOST LIKELY cause of the infant’s illness is A. Bordet Bordetella ella pertuss pertussis. is. B. Chlam Chlamyd ydia ia pneumo pneumonia niae. e. C. Moraxel Moraxella la catarrha catarrhalis. lis. D. Mycoplas Mycoplasma ma pneumo pneumonia niae. e. ANS: A Medical Microbiology by Jawetz., 23rd ed., page 282

224.

Which one of the following viruses establishes establishes a chronic persistent persistent infection infection in in humans? humans? A. Hepat Hepatiti itis s A virus virus B. Hepat Hepatiti itis s C viru virus s C. Herp Herpes esvi viru rus s D. Poli Poliov ovir irus us ANSWER: B Medical Microbiology by Jawetz., 23rd ed., page 475-476

225. 225.

Which Which one of the follow following ing viruses viruses is transmi transmitted tted via via the gastroi gastrointes ntestina tinall route? route? A. Delta-a Delta-asso ssociat ciated ed agent agent B. Hepa Hepati titi tis sA C. Hepa Hepati titi tis sB D. Hepa Hepati titi tis sC ANSWER: B Medical Microbiology by Jawetz., 23rd ed., page 474

226. 226.

A 33-year33-year-old old pregna pregnant nt woman woman deliver delivered ed a baby boy via via a normal vagin vaginal al delivery. delivery. Neither she nor the father has a history of genital or oral herpes. Although she had chickenpox as a child, she was exposed to a child with chickenpox about 10 days before delivery. There was no evidence of vesicular skin lesions on either the mother  or the child at the time of delivery. Three days after birth, the baby now develops vesicular skin lesions on his back, accompanied by a fever. Which one of the following viruses is MOST LIKELY to be the cause of the baby's infection? A. Cytom Cytomeg egalo alovir virus us B. Herpes Herpes simplex simplex virus virus type type 1 C. Herpes Herpes simplex simplex virus virus type 2 D. Huma Human n herpe herpesvi sviru rus s6 ANSWER: C Medical Microbiology by Jawetz., 23rd ed., page 434

227. 227.

Examinat Examination ion by dark-f dark-field ield micro microscop scopy y of scrapings scrapings from from a genital genital ulcer ulcer of 19-year 19-year old female reveals mobile, spiral-shaped organisms against a black background. The MOST LIKELY diagnosis is A. chancroid. B. Chlam Chlamyd ydia ia trach trachom omati atis s infect infection ion.. C. gonorrhea. D. syphilis. ANS: D Medical Microbiology by Jawetz., 23rd ed., page 333 39

228. 228.

Gram-p Gram-posit ositive ive cocci cocci are are observe observed d on a Gram stain stain of of purulent purulent disch discharge arge from from a skin skin lesion of a 7-year-old. Culture of the exudate reveals beta-hemolytic, catalase-positive organisms. Which one of the following laboratory tests is the MOST APPROPRIATE next step in the definitive identification of this organism? A. Bacitr Bacitrac acin in sens sensiti itivit vity y test test B. Bile ile solu solubi bili lity ty tes testt C. Coagulase ase te test D. Opto Optoch chin in sen sensi siti tivi vity ty tes testt ANS: C Medical Microbiology by Jawetz., 23rd ed., page 227

229. 229.

Which Which one of the the followin following g species species or serov serovars ars of Chlam Chlamydia ydia can can be transm transmitte itted d as a zoonotic infection? A. Chla Chlamy mydi dia a pneu pneumo moni niae ae B. Chla Chlamy mydi dia a psit psitta taci ci C. Chlam Chlamyd ydia ia tra tracho choma matis tis serova serovarr C D. Chlam Chlamyd ydia ia tra tracho choma matis tis serova serovarr D ANS: B Medical Microbiology by Jawetz., 23rd ed., page 365

230. 230.

Gram Gram stain of synovi synovial al fluid from from the knee knee of a 20-year-o 20-year-old ld woman woman with arthri arthritis tis reveals reveals Gram-negative cocci. To isolate the MOST LIKELY responsible agent, the specimen should be plated on A. Bord Bordet et-G -Gen engo gou u agar agar.. B. Chocolate agar . C. Eosin Eosin Meth Methyle ylene ne Blue Blue (EMB) (EMB) agar. agar. D. MacC MacCon onk key agar. gar. ANS: B Medical Microbiology by Jawetz., 23rd ed., page 709

231. 231.

The forma formation tion of colon colonic ic ulcers ulcers and and absesses absesses durin during g Shigella Shigella dysen dysentery tery is due due to A. ability of bacteria to survive stomach acidity because of the production of urease. B. bacteri bacterial al survival survival within within macrop macrophage hages. s. C. invasion of enterocytes and cell-to-cell spread of the bacteria. D. producti production on of of a superant superantigen igen by the the bacil bacillus. lus. ANS: C Medical Microbiology by Jawetz., 23rd ed., page 255-256

232. 232.

Which Which one of the the followi following ng organis organisms ms is the the MOST COMMO COMMON N cause cause of bacteri bacterial al traveler’s diarrhea? A. Camp Campyl ylob obac acte terr jeju jejuni ni B. ente enterot rotox oxige igeni nic c Esche Escheric richi hia a coli coli C. Salm Salmon onel ella la cho chole lera rasu suis is D. Shige igella lla son sonne neii ANS: B Medical Microbiology by Jawetz., 23rd ed., page 252

40

233. 233.

Dissemi Disseminati nation on of Salmonel Salmonella la typhi from from the intestine intestine to extrain extraintest testinal inal sites sites requires requires survival of this pathogen in which one of the following host cells? A. Colonic enterocytes B. Gastric epithelial cells C. M cells D. Macrophages ANS: D Medical Microbiology by Jawetz., 23rd ed., page 257-258

234. 234.

Gram Gram stain of cerebr cerebrospi ospinal nal fluid fluid from a 3-year3-year-old old child child with fever, fever, petech petechiae, iae, stiff stiff neck neck and positive Kernig and Brudzinski's signs reveals PMNs and a few Gram-negative cocci. The MOST LIKELY agent is A. Esch Escher eric ichi hia a coli coli.. B. Haem Haemop ophil hilus us infl influe uenz nzae ae type type b. b. C. Neis Neisse seri ria a menin meningi giti tidi dis. s. D. Streptoc Streptococcu occus s agalac agalactiae tiae (group (group B streptoc streptococci occi ). ANS: C Medical Microbiology by Jawetz., 23rd ed., page 302

235. 235.

Listeria Listeria monoc monocytog ytogene enes s infection infection of of an adult adult is MOST MOST LIKELY LIKELY to be acquir acquired ed by A. inge ingesti stion on of of cont contami amina nated ted food food.. B. inhala inhalatio tion n of contam contamina inated ted aero aerosa sals. ls. C. inoculat inoculation ion with a contami contaminate nated d needl needle. e. D. perso person-t n-to-p o-pers erson on conta contact. ct. ANS: A Medical Microbiology by Jawetz., 23rd ed., page 217

236. 236.

What What is the mechanis mechanism m of action action of of the antifu antifunga ngall drug fluco fluconaz nazole? ole? A. Binds to ergosterol ergosterol resulting in membrane membrane disruption disruption B. Inhibi Inhibits ts ergoster ergosterol ol synthe synthesis sis C. Inhibit Inhibits s protei protein n synthe synthesis sis D. Inhibit Inhibits s RNA RNA synthes synthesis is ANS: B Medical Microbiology by Jawetz., 23rd ed., page 652-653

237. 237.

A 10-yea 10-year-ol r-old d with with a dog dog complai complains ns of a “bump” “bump” on the the back back of of his head. head. He says says its its redness and itching have increased over the last three weeks. The lesion is several centimeters in diameter, with hair involvement and scaling. Examination of several of  his hairs in a KOH preparation reveals hyphae in and and around the hair shafts. What is your diagnosis? A. Acti Actino nomy myco cosi sis s B. Cand Candid idia iasi sis s C. Pityria Pityriasis sis versicol versicolor  or  D. Tine Tinea a capi capiti tis s ANSWER: D, Tinea capitis Medical Microbiology by Jawetz., 23rd ed., page 632

238. 238.

You are are assign assigned ed to a hospita hospitall in Phoeni Phoenix, x, Arizon Arizona. a. Several Several patie patients nts have have been been admitted in the past week with cough, chest pain, fever, and pneumonitis that has not responded to antibacterial therapy. You discover that most of the patients are new new arrivals to Arizona who had been working outdoors during the dusty wind storms that had occurred over the past past few weeks. The causative organism is observed in a KOH preparation of sputum. What is your presemptive diagnosis? 41

A. B. C. D.

Blas Blasto tomy myco cosi sis s Coccidi Coccidioido oidomyco mycosis sis Histo Histopl plasm asmos osis is Psit Psitta taco cosi sis s ANSWER: B, Coccidioidomycosis Medical Microbiology by Jawetz., 23rd ed., page 637-638

239. 239.

A 65-year-o 65-year-old ld male develops develops endocar endocarditi ditis s following following urinary urinary catheter catheterizat ization. ion. GramGrampositive, catalase-negative cocci are isolated from a blood culture. The MOST LIKELY agent is A. Enterococcus faecalis (group D streptococci) B. Stap Staphy hylo loco cocc ccus us aur aureu eus. s. C. Staph Staphylo ylococ coccu cus s epide epiderm rmidi idis. s. D. Streptoc Streptococcu occus s pyoge pyogenes nes (grou (group p A strep streptoco tococci) cci).. ANS: A Medical Microbiology by Jawetz., 23rd ed., page 235

240. 240.

A 22-year22-year-old old develop develops s an abcess abcess in his periton peritoneum eum follow following ing rupture rupture of his his appendix appendix.. Gram stain of the exudate from his foul-smelling abcess reveals numerous polymorphonuclear leukocytes, Gram-positive cocci, Gram-positive rods, and Gramnegative rods. Aerobic culture of the exudate at 37o C on blood and MacConkey agar  plates yields only enterococci (group D streptococci). The Gram-negative rods are MOST LIKELY A. Bact Bacter eroi oide des s frag fragil ilis is B. Esch Escher eric ichi hia a coli coli C. Pseu Pseudo domo mona nas s aeru aerugi gino nosa sa D. Shig Shigel ella la dyse dysent nter eria iae. e. ANS: A Medical Microbiology by Jawetz., 23rd ed., page 309

241.

Several people become ill within within two hours after leaving leaving a summer summer reunion reunion picnic. The majority of people complain of vomiting and diarrhea without fever, and recovered within 24 hours after the onset of symptoms. The contaminated food is determined to be a bowl of coleslaw prepared by someone with a stye on her left eyelid. Which one of the following is the MOST LIKELY source of the food poisoning? A. Clos Clostr trid idiu ium m botu botuli linu num m B. Clos Clostr trid idiu ium m perf perfri ring ngen ens s C. Stap Staphy hylo loco cocc ccus us aure aureus us D. Shige igella lla son sonne neii ANS: C Medical Microbiology by Jawetz., 23rd ed., page 226

242. 242.

A 35-year35-year-old old female female presen presents ts with fever, fever, headac headache he and a non-prod non-producti uctive ve cough. cough. The The patient is prescribed penicillin and sent home. Two weeks later, she returns with similar  symptoms. A Gram stain and routine culture of the patient's sputum reveal only normal mouth flora. Sputum is then cultured on specialized medium containing sterols, and, after two days, tiny colonies were visible under a microscope. Which one of the following organisms does the physician suspect is the cause of the patient's pneumonia? A. Coryn Coryneb ebact acteri erium um dipht diphther heriae iae B. Haem Haemop ophil hilus us influe influenz nzae ae type type b C. Legionella pneumophila D. Myco Mycopl plas asma ma pne pneum umon onia iae e

42

ANS: D Medical Microbiology by Jawetz., 23rd ed., page 345 243. 243.

A 70-year70-year-old old male, male, who is a chain-s chain-smok moker, er, complai complains ns of fever, fever, diarrhe diarrhea, a, cough cough and chest pain. Evidence of pneumonia is seen by chest X-ray. Due to the inability of the patient to produce a sputum specimen, a transtrachial aspirate is obtained. No respiratory bacterial pathogen is isolated from aspirated fluid cultured on blood agar  plates. However, poorly staining Gram-negative rods are isolated on buffered charcoalyeast extract (BCYE) agar after three days of incubation. The physician had already begun treatment with erythromycin due to the detection of bacilli in the aspirated fluid using a fluorescently-labeled monoclonal antibody specific for the causative agent. The MOST LIKELY pathogen responsible for this pneumonia is A. Haemop Haemophilu hilus s influen influenzae zae B. Legion Legionella ella pneumo pneumophi phila la C. Mycoplas Mycoplasma ma pneumon pneumoniae iae D. Neisseria meningitides ANS: B, Legionella pneumophila Medical Microbiology by Jawetz., 23rd ed., page 313

244. 244.

Which Which one one of the the follow following ing fung fungii exhibit exhibits s therma thermall dimorph dimorphism? ism? A. Aspergillus Aspergillus fumigatus fumigatus B. Candida tropicalis. tropicalis. C. Cryptococcus Cryptococcus neoformans D. Histoplasma Histoplasma capsulatum Answer: D Medical Microbiology by Jawetz., 23rd ed., page 640

245.

An otherwise otherwise healthy 35-year-old 35-year-old female female florist florist presented presented to her her primary primary care care physician physician complaining of a persistent lesion on the third digit of her left hand. This had been present for several weeks and showed no sign of resolving. Upon examination, the patient was found to have a mild fever (1000F), the aforementioned lesion and two nodules on her left forearm. A biopsy of the lesion was performed, and the pathology report indicated the presence of numerous polymorphonuclear leukocytes and a rare elongated budding yeast. What is the MOST LIKELY etiologic agent? A. Blas Blasto tomy myce ces s derm dermat atit itid idis is B. Spor Sporot othr hrix ix sche schenc ncki kiii C. Hist Histop opla lasm sma a caps capsul ulat atum um D. Noca Nocard rdia ia aste astero roid ides es Answer: B Medical Microbiology by Jawetz., 23rd ed., page 632-633

246. 246.

Mode of trans transmis mission sion of Trich Trichomo omonas nas vaginal vaginalis is is: is: A. Inge Ingest stio ion n of inf infes este ted d beef  beef  B. Skin Skin con conta tact ct with with larv larvae ae C. Mos Mosquito ito bit bites D. Sexual ANS:D Medical Microbiology by Jawetz., 23rd ed., page 663-664

247.

Mode of transmission of Enterobius of  Enterobius vermicularis is: A. Sexual B. Blac Blackf kfly ly bit bites es C. Lous Louse e bite bites s D. Feca Fecall-or oral al

43

Ans. D Medical Microbiology by Jawetz., 23rd ed., page 689 248.

Oxygen requirements of Neisseria of  Neisseria gonorrhoeae is: A. Aero erobic B. Facu Faculta ltativ tive e anaero anaerobe be C. Microa Microaer eroph ophili ilic c D. Oblig Obligate ate ana anaero erobe be Ans. A Medical Microbiology by Jawetz., 23rd ed., page 295

249.

Mode of transmission of Borrelia of  Borrelia burgdorferi is: A. Direct Direct conta contact ct with with animals animals B. Coni Conidia dia inhala inhalatio tion n C. Sexual D. Tick Tick bite bites s Ans. D Medical Microbiology by Jawetz., 23rd ed., page 336

250.

Therapy of Vibrio of  Vibrio cholerae is: A. TMP-SMX B. ORS C. Ceft Ceftri riax axon one e D. Eryt Erythr hrom omyc ycin in Ans. B Medical Microbiology by Jawetz., 23rd ed., page 271

251.

Family of Measles of Measles virus is: A. Param Paramyx yxovi ovirid ridae ae B. Rhab Rhabdo dovi viri rida dae e C. Ortho Orthomy myxo xovir virida idae e D. Flav Flaviv ivir irid idae ae Ans. A Medical Microbiology by Jawetz., 23rd ed., page 550

252.

Family of Hepatitis of Hepatitis A virus is: A. Papo Papova vavi viri rida dae e B. Hepa Hepadn dnav avir irid idae ae C. Parv Parvov ovir irid idae ae D. Pico Picorna rnavir virid idae ae Ans. D Medical Microbiology by Jawetz., 23rd ed., page 466

253.

Phenotypic characteristic of  Streptococcus pyogenes is: A. Alph Alpha-h a-hem emoly olytic tic B. Oxida xidase se "+" "+" C. H2S "-" D. BetaBeta-he hemo molyt lytic ic Ans. D Medical Microbiology by Jawetz., 23rd ed., page 235

44

254.

Mode of transmission of Rickettsia of  Rickettsia prowazekii is: A. Drop Drople lett nucl nuclei ei B. Direct Direct contact contact with with anima animals ls C. Lous Louse e bit bites es D. Woun Wound d infe infect ction ion Ans. C Medical Microbiology by Jawetz., 23rd ed., page 350

255.

Phenotypic characteristic of  Gonococcus is: A. Fastidious B. Mycolic acid in cell wa wall C. Catalase "-" D. Non-lactose fe fermenter  Ans. A Medical Microbiology by Jawetz., 23rd ed., page 295

256. 256.

Staphylo Staphylococc coccal al food food poisonin poisoning g is the result result of  A. a heat heat lab labil ile e ente entero roto toxi xin. n. B. ingesti ingestion on of a pre-formed pre-formed entero enterotox toxin in produce produced d by Staphylo Staphylococc coccus us aureus during bacterial multiplication in contaminated food. C. ingestio ingestion n and multiplica multiplication tion of coagulas coagulase-n e-negat egative ive Staphylo Staphylococc coccus us sp. D. multipl multiplicat ication ion of Staphy Staphyloco lococcu ccus s aureus aureus in the smal smalll intestine intestine.. ANS: B Medical Microbiology by Jawetz., 23rd ed., page 225

257.

A 19-year-ol 19-year-old d soldier soldier develops develops fever, carditis, and migratory migratory polyarthrit polyarthritis. is. While While taking taking her history, the patient remembers that she had a "sore throat" three weeks previously. A throat culture of the patient yields normal flora only. The laboratory result MOST CONSISTENT with these findings is that the patient has high titers of antibody to: A. menin meningoc gococc occal al lipoo lipoolig ligos osacc accha harid ride. e. B. pneum pneumoc ococc occal al IgA IgA protea protease se.. C. pneum pneumoc ococc occal al polys polysacc accha harid ride. e. D. stre strept pto olysi lysin n O. ANS: D Medical Microbiology by Jawetz., 23rd ed., page 234

258. 258.

A 65-year65-year-old old man man with a colonic colonic tumor tumor undergoe undergoes s a bowel resec resection tion.. Three days days later, later, the patient develops a fever, and the drainage from the surgical site is foul smelling. The skin near the site of the drainage is markedly erythematous. Gram stain of the drainage reveals Gram-positive cocci and Gram-negative rods. Aerobic culture of the drainage yields streptococci. The Gram-negative rod seen on Gram stain was most likely: A. Bact Bacter eroi oide des s frag fragil ilis is B. Clos Clostr trid idiu ium m perf perfri ring ngen ens s C. Esch Escher eric ichi hia a coli coli D. Pseu Pseudo domo mona nas s aeru aerugi gino nosa sa ANS: A Medical Microbiology by Jawetz., 23rd ed., page 306

259. 259.

A 50-year50-year-old old rancher rancher scrap scrapes es his hand hand with clipper clippers s while sheari shearing ng sheep. sheep. Two Two days later he notices an ulcer surrounded by vesicles at the site of the wound. Eight days after the injury, the wound develops a black necrotic center (eschar) surrounded by

45

edematous fluid. The rancher experiences mild systemic symptoms. The most likely cause of the infection is: A. Baci Bacill llus us anth anthra raci cis. s. B. Clos Clostr trid idiu ium m teta tetani ni.. C. Neis Neisse seri ria a meni mening ngit itid idis is.. D. Streptoc Streptococcu occus s pyoge pyogenes nes (grou (group p A strep streptoco tococci) cci).. ANS: A Medical Microbiology by Jawetz., 23rd ed., page 202-203 260. 260.

A 19-year19-year-old old colleg college e sophomor sophomore e dies 24 hours hours after after the the develo developmen pmentt of fever, fever, petechiae, purpura, and disseminated intravascular coagulation. Culture of  cerebrospinal fluid and blood obtained from the patient on admission to the hospital MOST LIKELY yielded: A. Gram-neg Gram-negativ ative, e, lactose lactose ferment fermenting ing rods. rods. B. Gram-ne Gram-negat gative, ive, oxidase oxidase positiv positive e diplo diplococ cocci. ci. C. Gram-pos Gram-positiv itive, e, catalase catalase negativ negative e cocci. cocci. D. Gram-pos Gram-positiv itive, e, catalase catalase positiv positive e cocci. cocci. ANS: B Medical Microbiology by Jawetz., 23rd ed., page 302

261.

The blood blood culture culture of a patient with a presumptive presumptive diagnosis diagnosis of endocarditis endocarditis yields yields nonnonhemolytic colonies of Gram-positive cocci. The isolate is catalase negative, can grow in 6.5% NaCl, and is resistant to penicillin. The isolate is MOST LIKELY: Enterococci . A. Enterococci. B. Streptoc Streptococcu occus s agalac agalactiae tiae (group (group B streptoc streptococci occi). ). C. Stre Strept ptoc ococ occu cus s bovi bovis. s. D. Streptoc Streptococcu occus s mutan mutans s (Virid (Viridans ans streptoc streptoccoci coci). ). ANS: A Medical Microbiology by Jawetz., 23rd ed., page 233

262. 262.

A 25-year25-year-old, old, previo previousl usly y healthy healthy woman woman develop develops s fever fever within within 24 hours of delive delivery ry of  an infant born 6 weeks prematurely. Blood culture of the mother reveals Gram-positive cocci that are catalase negative, beta-hemolytic on blood agar, and resistant to bacitracin. The MOST LIKELY cause of the bacteremia is: A. Ente Entero roco cocc cci. i. B. Staphyl Staphylococ ococcus cus aureus. aureus. C. Streptococcus Streptococcus agalactiae agalactiae (group (group B streptococ streptococci). ci). D. Strepto Streptococ coccus cus pneum pneumonia oniae. e. ANS: C Medical Microbiology by Jawetz., 23rd ed., page 233

263.

An HIV-infec HIV-infected ted homosex homosexual ual man man presents presents with Kaposi's sarcoma. sarcoma. Which Which viral genome genome is MOST LIKELY to be present in his tumor biopsy? A. Herp Herpes es sim simpl plex ex vir virus us tup tupe e1 B. Huma Human n cyto cytome mega galv lvir irus us C. Huma Human n her herpe pesv svir irus us type type 6 D. Huma Human n her herpe pesv svir irus us typ type e8 Answer: D Medical Microbiology by Jawetz., 23rd ed., page 450

46

264.

Which one of the following viral infections infections is not effectively effectively prevented prevented by vaccination? vaccination? A. Hepa Hepati titi tis s B viru virus s B. Poliovirus C. Rhinovirus D. Rubella lla vir virus Answer: C Medical Microbiology by Jawetz., 23rd ed., page 543

265. 265.

Prions Prions cause cause spongifo spongiform rm enceph encephalop alopathi athies es includin including g the human human disease disease A. AIDS AIDS deme dement ntia ia comp comple lex. x. B. Creu Creutzf tzfel eldt dt-Ja -Jaco cob b dise diseas ase. e. C. scrapie. D. subacut subacute e sclero sclerosing sing panence panencepha phalopa lopathy thy (SSP (SSPE). E). Answer: B Medical Microbiology by Jawetz., 23rd ed., page 584

266.

Congenital Congenital infection infection by which which one one of the following following viruses can result result in in birth birth defects? defects? A. Adenovirus B. Cyto Cytome mega galo lovi viru rus s C. Hepa Hepati titi tis s B viru virus s D. Huma Human n imm immuno unode defic ficien iency cy virus virus Answer: B Medical Microbiology by Jawetz., 23rd ed., page 443

267. 267.

Viral Viral carditi carditis s of adoles adolescen cents ts and and adults adults is frequen frequently tly cause caused d by A. Coxsackieviruses . B. Orth Orthom omyx yxov ovir irus uses es.. C. Pseu Pseudo dora rabi bies esvi viru rus. s. D. Reoviruses. Answer: A Medical Microbiology by Jawetz., 23rd ed., page 495

268.

Several newborns newborns housed housed in a neonatal neonatal intensive intensive care unit have have developed developed nosocomial nosocomial pneumonia. Most are wheezing and some have required care such as mechanical ventilation, increased doses of oxygen, antibiotic therapy, or monitoring for apnea. A simple, rapid diagnostic test reveals a common viral antigen in nasal wash specimens from each of the patients. The virus MOST LIKELY to be responsible is A. Coronavirus B. Echovirus B2 B2 C. Resp Respira irator tory y syncy syncytia tiall virus virus D. Rube Rubeol ola a vir virus us (mea (measl sles es)) Answer: C Medical Microbiology by Jawetz., 23rd ed., page 558-559

269. 269.

A 7 year-old year-old newly newly adop adopted ted Russia Russian n child child who has been been in the the United United States States 10 days days presents with fever, malaise, sore throat, and a grey membrane on the tonsils, uvula, and soft palate. The child's immunization history is unknown. The physician obtains a swab of the child's throat which provokes bleeding of the membrane. The swab is sent to the laboratory for routine throat culture. Two days later the laboratory reports only the presence of normal flora in the specimen. The MOST LIKELY organism responsible for  this child's illness is: A. Bord Bordete etella lla pertus pertussis sis.. B. Coryne Corynebac bacteri terium um diphtheri diphtheriae. ae. 47

C. Haemop Haemophilu hilus s influenz influenzae ae type type b. D. Listeri Listeria a mono monocyto cytogene genes. s. Answer: B Medical Microbiology by Jawetz., 23rd ed., page 214 270. 270.

Soft chees cheese e imported imported from from Mexico Mexico was was implica implicated ted as the the vehicle vehicle in an outbr outbreak eak of  meningitis and bacteremia that occurred among attendees of a large company picnic. Twenty of the 40 affected individuals were pregnant women. One of the victims had AIDS. Gram positive, non-spore forming rods were isolated from the cheese that had been stored in the cold. The MOST LIKELY organism responsible for this outbreak is: A. Baci Bacill llus us cere cereus us B. Clost Clostrid ridium ium diffi diffici cile le C. Clostri Clostridium dium perfring perfringens ens D. Listeri Listeria a monocy monocytog togenes enes Answer: D Medical Microbiology by Jawetz., 23rd ed., page 217

271. 271.

A 25 year year old patient patient prese presents nts at a sexuall sexually y transmi transmitted tted disea disease se (STD) (STD) clinic clinic with with a painful genital ulcer. On examination, the ulcer does not appear to be typical of herpes simplex virus. Dark field microscopy of exudate from the ulcer is negative for spiralshaped organisms. However, a small, gram negative rod is isolated from culture of the exudate on chocolate agar. The organism MOST LIKELY to be responsible for the lesion is: A. Chlamy Chlamydia dia trachom trachomatis atis.. B. Haemop Haemophilu hilus s ducreyi ducreyi.. C. Neisser Neisseria ia gonorrho gonorrhoae. ae. D. Trep Trepone onema ma pallid pallidum um.. Answer: B Medical Microbiology by Jawetz., 23rd ed., page 281

272. 272.

Which Which ONE ONE of the the follow following ing disea diseases ses is is transm transmitte itted d by lice? lice? A. Epide Epidemic mic typhu typhus s B. Ehrl Ehrlic ichi hios osis is C. Psit Psitti tico cosi sis s D. Scru Scrub b typh typhus us Answer: A Medical Microbiology by Jawetz., 23rd ed., page 350

273.

A young young army army recruit recruit presents presents at a sexually transmitted transmitted disease disease (STD) clinic with acute acute urethritis. A Gram-stained preparation of his urethral exudate reveals neutrophils with intracellular Gram-negative diplococci. The patient is treated with ceftriaxone and sent home. He is requested to return in one week so that a urethral culture can be obtained to test for proof of antibiotic cure. Which ONE of the following culture media should be used for the follow-up culture? A. Blood lood agar  agar  B. Bord Bordet et Gen Gengou gou agar  agar  C. MacCon MacConke key y agar  agar  D. Thaye Thayer-M r-Mart artin in agar  agar  Answer: D Medical Microbiology by Jawetz., 23rd ed., page 295

48

274. 274.

The growth growth in in the vagina vagina of which which ONE ONE of the follo followin wing g bacteria bacteria helps helps maint maintain ain a protective acid pH? A. Bact Bacter eroi oide des s B. Gard Gardne nere rell lla a C. Lacto Lactoba bacil cillus lus D. Mobi Mobilu lunc ncus us Answer: C Medical Microbiology by Jawetz., 23rd ed., page 307

275.

A 25 year-old sexually active woman complains complains of vaginal itching, odor, and discharge discharge.. The vaginal walls are red and slightly swollen, but the cervix is normal. A sample of her  vaginal discharge is tested with 10% KOH, which produces a strong fishy odor. Upon microscopic examination of the discharge, you find clue cells but no pear-shaped flagellates or pseudohyphae. A Gram stain reveals numerous Gram-negative curved rods. What is your diagnosis? A. Bacte Bacteria riall vagi vagino nosis sis B. Pelvic Pelvic inflam inflammat matory ory disea disease se (PID) (PID) C. Syph Syphil ilis is D. Trich Trichom omon onias iasis is Answer: A Medical Microbiology by Jawetz., 23rd ed., page 316

276. 276.

A 25 year-ol year-old d male deve develops lops an an ulcerati ulcerative ve lesion lesion of the the hand and and severa severall fluctuant fluctuant nodules along the lymphatics draining that site. The patient is seen in the dermatology clinic at the Naval Hospital. Case history reveals that he is a weekend gardener who cultivates roses as a hobby. What is the MOST LIKELY fungal disease? A. Chrom Chromob oblas lastom tomyc ycos osis is B. Myce ycetoma oma C. Phaeo Phaeohy hypho phomy mycos cosis is D. Sporo Sporotri trich chos osis is Answer: D Medical Microbiology by Jawetz., 23rd ed., page 633

277. 277.

A 50 year-old year-old farmer farmer from from Sorsogo Sorsogon n who is fond of eating eating fresh freshwate waterr crabs is suffer suffering ing from chronic pneumonia with hemoptysis. Which parasite etiology is MOST likely? A. Capi Capill llar aria ia phili philipp ppin inen ensi sis s B. Necator americanus C. Para Parago goni nimu mus s west wester erma mani ni D. Schi Schist stos osom oma a jap japon onic icum um

278.

Knowing the life cycle of Schistosoma japonicum, a rural health doctor is now tasked with educating the population of a district in Davao del Sur about prevention of this disease. Which specific developmental stage of this parasite should be avoided to prevent initial infection? A. cercaria B. metac tacerc ercaria aria C. miracidium D. schis chisto tos somu omule

279. 279.

In relation relation to questi question on #72 above, above, which which of the the following following measure measures s would would this doctor  doctor  teach is most practical for preventing S. japonicum infection? A. avoid avoid eating eating raw fresh fresh wat water er sna snails ils B. boil boil dri drinki nking ng wate waterr for for 20 minut minutes es C. clean clean and and cook cook vegeta vegetable bles s wel welll D. wear wear protec protective tive high high boot boots s while while wading wading in in streams streams

49

280. 280.

A preventi preventive ve measu measure re against against paras parasites ites is is footwear footwear.. Which Which of these these paras parasites ites enter enters s through the skin and its infective stage is a filariform larva? A. Neca Necato torr amer americ ican anus us B. Trichinella spiralis C. Trichuris trichiura D. Ente Entero robi bius us ver vermi micu cula lari ris s

281. 281.

Some Some protozo protozoans ans have have orga organell nelles es which which enab enable le them them to move around. around. Which Which of the the following species possess cilia as motility organelles? A. Balan Balantid tidiu ium m coli coli B. Entam Entamoe oeba ba histoly histolytic tica a C. Giardia lamblia D. Plas Plasmo modiu dium m malar malariae iae

282. 282.

Ten year-o year-old ld Theira Theira has been been having having vague perium periumbilic bilical al abdomina abdominall discomfo discomfort rt for  three months. She does not have diarrhea. Her fecalysis reveals round golden brown tri-layered eggs with cortication. What is the causative agent of her abdominal infection? A. Asca Ascari ris s lumb lumbri rico coid ides es B. Bala Balant ntid idiu ium m coli coli C. Taenia saginata D. Tric Trich huris uris tric trichi hiur ura a

283. 283.

Mrs. Sintas Sintas compl complains ains to her her family family doctor doctor that she she has been been having having vague abdom abdominal inal pains and bloody diarrhea for the past two days. She had recently arrived from a business trip to Leyte. Which of the following intestinal parasitic diseases is LEAST LIKELY to present with bloody diarrhea? A. Amebiasis B. Giardiasis C. Trich ichuria urias sis D. Stro Strong ngyl yloi oidi dias asis is

284.

Adult stages of intestinal intestinal parasites parasites have their own way way of establishing establishing themselves themselves in their chosen habitat. Which of these species has a long anterior end which penetrates the mucosa to obtain tissues for its nutrition? A. A. Asca Ascari ris s lum lumbr bric icoi oide des s B. B. Enter Enterob obius ius verm vermicu icular laris is C. Strongyloides stercoralis D. Tric Trichu huri ris s tric trichi hiur ura a

285.

Except for the Schistosome species, all trematodes have similar life cycles. The infective stage, for example is which of these developmental stages for the liver, lung, and intestinal flukes? A. cercaria B. meta etacerca rcaria ria C. redia D. sporocyst

286. 286.

Hydatid Hydatid Disea Disease se is endemic endemic in in small small areas areas of Souther Southern n Luzon Luzon and Bicol Bicol region region where where sheep are raised. What is the causative parasite of this disease? A. Ancy Ancylo lost stom oma a duo duode dena nale le B. Brugi rugia a malayi ayi C. Echi Echino noco cocc ccus us granu granulo losu sus s D. Fasc Fascio iolo lops psis is bus buski ki

50

287.

The trophozoite stage of this protozoa has rbc as inclusions in the cytoplasm: (p. 121) A. Enta ntamoeb oeba co coli B. Enta Entamo moeb eba a his histo toly lyti tica ca C. Endolimax nana D. Enta Entamo moeb eba a hart hartma mani niii

288. 288.

One of of the follow following ing is not not a pseudop pseudophyll hyllidea idean n tapeworm tapeworm.. (p. 513) 513) A. Diph Diphyl yllo lobo both thri rium um latu latum m B. Ligu Ligula la inte intest stin inal alis is C. Sparganum proliferum D. Taenia enia sol solium

289. 289.

Bachma Bachman n intraderm intradermal al test is is a highly highly specific specific test test for this this disease. disease. (p. (p. 239) 239) A. Ascariasis B. Enterobiasis C. Trichinosis D. Amoebiasis

290. 290.

One of the the followin following g is not a common common featur feature e of uncompl uncomplicat icated ed P. falcip falciparum arum in in peripheral smear: (p. 186) A. appl appliq iqué ué for form m of of para parasi site te B. multi multipl ple e inf infect ection ion of one rbc rbc C. pres presen ence ce of of Maur Maurer er’s ’s cle cleft fts s D. roun rounde ded d micro microga game meto tocy cyte te

291.

A 10-year-old 10-year-old girl, with perianal perianal pruritus, pruritus, was brought by her her mother mother to to her pediatrician. pediatrician. What is your most probable diagnosis? A. Ascariasis B. Trich ichuria urias sis C. Entero erobiasi asis D. Hook Hookw worm orm infe infect ctio ion n Ref. Clinical Parasitology by Beaver and Jung 9 th edition, pp. 304-305 MPL: 0.75

292. 292.

A 40-year 40-year old male male overse overseas as worker worker compl complaini aining ng of muscle muscle pains pains seek seek medica medicall attention upon his arrival to the Philippines. Biochemical tests showed elevated creatinine, phosphokinase, lactate dehydrogenase and myokinase levels. Results of  complete blood count showed high blood eosinophilia. What is your most probable diagnosis? A. Taeniasis B. Capilla illarrias iasis C. Trichinosis D. Filariasis Ref. Clinical Parasitology by Beaver and Jung 9 th edition, pp.238-239 MPL: 0.5

293.

Sexually active 26-year old woman woman consulted consulted a VD control control clinic for vaginal vaginal itching itching and purulent discharge. Your tentative diagnosis should include the following: A. gonor onorrh rhea ea B. tric tricho homo moni nias asis is C. cand candid idia iasi sis s D. all all of of the the abov above e Ref. Clinical Parasitology by Beaver and Jung 9 th edition, pp. 50 MPL: 0.25

51

294. 294.

A 30-year 30-year old woma woman, n, native native of Bulaca Bulacan, n, traveled traveled to to Palawan Palawan for for a vacation. vacation. After After 2 weeks she consulted her doctor with chills and fever as her main complains. What parasitologic test would you request to establish your diagnosis? A. sput sputum um exa examin minat ation ion B. stoo stooll exa examin minat ation ion C. thick thick blood blood sme smear  ar  D. thick thick and and thin thin blood blood smear  smear  Ref. Clinical Parasitology by Beaver and Jung 9 th edition, pp. 180-181 MPL: 0.5

295. 295.

A 6-month 6-month old baby baby was was brought brought in for for a fever fever of one-day one-day durati duration. on. She She is found found to have have a temperature of 38.5°C and with hepatomegaly (3 cm) and splenomegaly (3 cm). Microscopic examination revealed infected RBC with presence of band form stages. What is your most probable diagnosis? A. Not ma malaria B. Plasmodium falciparum C. Plasm lasmod odiu ium m viva vivax x D. Plas Plasmo modi dium um mala malari riae ae Ref. Clinical Parasitology by Beaver and Jung 9 th edition, pp. 184-185 MPL: 0.75

296. 296.

Hepatos Hepatosplen plenome omegaly galy is the the hallmark hallmark of this this disease disease which which is endemic endemic in Leyte, Leyte, Samar  Samar  and Mindoro. This is caused by: A. Schi Schist stos osom oma a jap japon onic icum um B. Echi Echino nost stom oma a iloc ilocan anum um C. Pila ila luz luzonica ica D. Capi Capill llar aria ia phili philipp ppin inen ensi sis s Reference: Brown & Neva, 1983. Basic Clinical Parasitology . 5th Edition. The McGraw Appleton-Century Crofts. p. 241

297. 297.

Pruritus Pruritus vulvae vulvae is a characte characterist ristic ic symp symptom tom in : A. Trichinosis B. Tric richomon monias iasis C. Ente nterobia biasis D. Scabies Reference: Brown & Neva, 1983. Basic Clinical Parasitology . 5th Edition. The McGraw Appleton-Century Crofts. p 46

298. 298.

Finding Finding banan bananaa- or sausa sausagege-shap shaped ed gameto gametocyte cytes s in periph periphera erall smear smear is a unique unique characteristic of: A. Plas Plasmo modi dium um mala malari riae ae B. Plasm lasmod odiu ium m viva vivax x C. Plas Plasmo modi dium um falc falcip ipar arum um D. Plasm lasmod odiu ium m oval ovale e Reference: Brown & Neva, 1983. Basic Clinical Parasitology . 5th Edition. The McGraw Appleton-Century Crofts. p. 81-82

52

299.

Wearing protective protective footwear footwear would would NOT NOT prevent prevent transmiss transmission ion of which of the following parasite? A. Ancylostoma B. Leptospira C. Trichuris D. Schistosoma Reference: Brown & Neva, 1983. Basic Clinical Parasitology . 5th Edition. The McGraw Appleton-Century Crofts. pp. 126

300. 300.

Taenia Taenia solium solium and Trichi Trichinell nella a spiralis spiralis can be preve prevented nted by eating eating prope properly rly cooked cooked:: A. fish B. pork C. beef   D. vegetables Reference: Brown & Neva, 1983. Basic Clinical Parasitology . 5th Edition. The McGraw Appleton-Century Crofts. pp. 185, 111

301.

Recommendatio Recommendation n for the control control of of hookworm hookworm in endemic endemic areas include the constructio construction n of sanitary facilities and : A. wear wearin ing g of foo footw twea ear  r  B. reduc reductio tion n of the the stra stray y cats cats popu populat lation ion C. throu through gh coo cookin king g of meat meats s and and fish fish D. use of of insecti insecticide cides s to contro controll mosquit mosquitoes oes and and flies. flies. Reference: Brown & Neva, 1983. Basic Clinical Parasitology . 5th Edition. The McGraw Appleton-Century Crofts. p. 126

302. 302.

Humans Humans becom become e infected infected when when they they swallow swallow the eggs eggs and and develop develop cysts cysts in in various various organs to cause hydatidosis. This tapeworm is could be best prevented by: A. reducing reducing larval larval E. multilo multilocula cularis ris in in sheep sheep and hogs B. reducin reducing g adult adult E. granulo granulosus sus infectio infection n in in dogs dogs C. prope properr and sanit sanitary ary dispos disposal al of waste waste D. surg surgic ical al inte interv rven enti tion on Reference: Brown & Neva, 1983. Basic Clinical Parasitology . 5th Edition. The McGraw Appleton-Century Crofts. p. 197

303.

Dyspnea Dyspnea in Loeffler’s syndrome syndrome caused caused by larval invasion invasion in massive massive ascariasis ascariasis in in the lungs could be best treated with: A. salb salbut utam amol ol neb nebul uliz izat atio ion n B. giving giving mebend mebendazol azole e to treat treat the the infectio infection n C. both A and B D. neithe itherr A nor nor B Reference: Brown & Neva, 1983. Basic Clinical Parasitology . 5th Edition. The McGraw Appleton-Century Crofts. Pp. 134-135

53

View more...

Comments

Copyright ©2017 KUPDF Inc.
SUPPORT KUPDF